Vous êtes sur la page 1sur 36

CRIMINAL LAW OUTLINE GOLDMAN FALL 2013

Start of a Criminal Case

Prosecutor shall only press charges when he knows or should know that the charges are supported by probable
cause.
Grand Jury (indictment decision is approved by a jury ex parte when they probable cause that the crime was
committed and that the person sought to be charged committed it) vs. Preliminary Hearing (review of charges
pressed directly by the prosecutor in front of judge in the presence of defendant)

Punishment
(a) What is punishment? Negative consequences imposed for breaking/violating rules. Not every action of the
government is a punishment, the government can revoke a privilege and that is not punishment, or can order civil
commitment, and sometimes thats not punishment either because it does not implicate some of the objectives of
criminal punishment. To determine whether criminal or civil, legislature uses 2 standards: Purpose (the purpose of
the act has to be civil) and Impact (if too punitive, it would negate its civil nature).
Kansas v. Hendricks, where reoccurring pedophile was put in civil commitment when he was about to be
released from jail. Court said was not double jeopardy because civil commitment was not for retribution or
deterrence. (Followed in Florida)
(b) Theories of Punishment (objectives)
1. Retribution seeks to punish a criminal because he deserves it (looking back)
2. Incapacitation seeks to reduce future crime by depriving the individual from the ability of recommitting
(looking forward)
3. Deterrence notion of reducing crime through fear of punishment (looking forward)
- General to deter the public from committing similar crimes
- Special to deter the individual from committing the same crime
4. Denunciation to express publicly societal disapproval of the conduct, seeks to educate society
(c) Constitutional Limits on Punishment:
Ex Post Facto Laws: Laws passed after the occurrence of an eventwhich retrospectively changes the
legal consequences of that eventcan only be applied when beneficial to the defendant.
Bills of attainder: Laws cannot target a specific group or person.
Due Process Clause: Cant criminalize a person without fair notice. If a statute is vague there is no fair
notice.

Making Criminal Law


(a) Legislative supremacy in criminal law criminal laws are made by the legislature, not by judges and
prosecutors. Common law has been abandoned by most jurisdictions. Only used to aid interpretation.
(b) Sources of Criminal Law:
Common Law: Judge made law (case law). We look at this to see if a common law crime applies or
interpretation can help interpret a statute.
- If someone is convicted of a common law crime the most they could be convicted for is a
misdemeanor.
State & Federal Statutes: Codified through the legislature.
Model Penal Code: drafted by the legal scholars (thru American Law Institute) to be used as a guide by
legislators reforming their state's criminal laws.
Reception Statutes: The adoption in whole or in part of the law of one jurisdiction by another jurisdiction.
-

Ex. In Florida, Where no Florida statute applies, common law of England is applied but not as to
the modes or degrees of punishment. Common law crimes can only be misdemeanors, punished by
fines or minimal imprisonment.

Page 1 of 36

(c) Keller v. Superior Court: D (Husband) found out wife was pregnant by another dude. Stopped wife to confirm,
and beat her in the stomach until killing fetus. Ct determined that D could not be tried for homicide because
homicide of a fetus did not exist at common law because it was not considered to be a human being. This case
gave us 2 principles:
o Common law terms: If undefined or unclear it will use its common law meaning in the absence of a
contrary legislative intent.
o Rule of Lenity: If a statute has multiple possible interpretations, the court must apply the one that favors
the defendant. (tie breakers only or only if court cant decide the meaning of the statute after applying all of
the interpretation rules.) **Recognized by FL but not MPC**
(d) Statutes can be Void for Vagueness: If a statute is ambiguous the court will not enforce it.
1. Facially void- the court has found that after using all the interpretive rules they still cant make sense out of
it. Its void in all its possible applications foreverinapplicable to anyone.
Ex.: City of Chicago v. Morales: A city ordinance punished people for loitering in the streets for
any purpose, even an innocent person. Ct found that failure to give notice to a defendant through
the ordinance of the specific conduct that constituted a crime, or its purpose, was found to be
unconstitutionally a violation of due process.
2. Void as applied- the statute looked good until tested by the case at bar, and in its application to that case it
makes no sense thus voidinapplicable only to a specific defendant.
(e) Statutory Interpretation Three Schools of Thought (statutes are given its plain & ordinary meanings, but if
they are ambiguous, then the court can apply one of the following:
a. Textualism
i. Go strictly to the text & what the words actually mean (four corners)
ii. Scalia says to look at other statutes of that time.
b. Intentionalism
i. Look at what the drafters of the statute intended when drafting the law at the time the law was
passed.
ii. Laws should be construed as to not to lead to injustice, oppression or an absurd consequence.
c. Dynamic Statutory Interpretation
i. Sometimes the court will take a textualist and intentionalist view in addition to looking at
constitutional developments and current societal facts.
(f) Elements of a Crime (M.A.C.) (proven beyond a reasonable doubt by prosecutor)
a. Conduct (Actus Reus)*
b. Mental State (Mens Rea)*
c. Causation (usually in homicide crimesbut for & proximate cause)
(* actus reas and mens rea must occur at the same time)

Conduct
(a) General Rule: A person is not ordinarily guilty of a criminal offense unless his conduct includes a voluntary act.
**Followed by Common law, FL, MPC, and Non-MPC states**
A person is never prosecuted solely for his thoughtsthere must be some externality of those thoughts.
Voluntary Act: A volitional muscular contraction or bodily movement by the actor based on his effort and
determination. (movement can be slight and volitional means controlled by the mind of the actor-- Can
include speech because the mouth, tongue and similar body parts are involved; and habitual acts.)
o Except: An act is non-volitional when the act is solely controlled by the brain, such as epilepsy
seizures, reflexes, unconscious movements, and hypnosisthus no criminal liability attaches. (Ex.
Woman that snores loudly and disturbs neighbor.)
Status: Crimes that punish a person for being rather than doingconstitutes cruel and unusual punishment
in violation of the 8th amendment of the Constitution. But it is constitutional to punish manifestation of
addiction.
o Robinson v. California: California statute declared the use and addiction to Narcotics a crime. D
was arrested for having healed scars on his arm from prior use. The court held that people could
become addicted both under innocent and voluntary circumstances. There must be an act that is
voluntary, even if the addition itself earlier on was voluntary, at the present moment when D was
Page 2 of 36

arrested it was not a crime but an illness (status). Illnesses cannot be punished. D could have used
the narcotics out of the State of Cali or years ago, yet under their statute he is still being punished
for being and not for acting.
o Powell v. Texas: Texas prohibited getting drunk and being found as such in a public place. At trial
D argued that punishing him was unconstitutional because under Robinson he is an addict. The
court held that the punishment was not for being an alcoholic but rather for being as such in a
public place, which got him there via a voluntary act.
o Martin v. State: Martin was drunk and at home. Police arrested him and took him onto the public
highway where D curses at the police. D is charged with being drunk in public and cursing at the
cops. The court held that the act of being drunk occurred in his home and not in public. He was
then involuntarily taken onto a highway in his condition thus not satisfying the elements of the
charge.
Condition: a state in which you are and cannot be changed. (ie. Homelessness)
Coerced Acts: An act is not involuntary merely because the individual is compelled to perform the act.
(i.e. Duress. Can be used as affirmative defense.
o Ex. A straps a bomb to Bs chest and tells B to rob a bank. B does it. Bs actus reus was voluntary.
Although mens rea is not satisfied, actus reus is.
Time Framing- The prosecution does not need to show that every act or even that the last act of the
accused was voluntary in order to establish criminal liability. It is sufficient that the defendants conduct
included a voluntary act at some point.
o People v. Decina: D drove knowing he suffered from seizures. The court held that since D knew
that he was susceptible to random attacks, his act was reckless. The answer of the court in this case
depended on time framing. If the court had constructed a narrow time frame, specifically the
conduct that the instant the car hit the victims Ds conduct would have not involved a voluntary act.
While a broader time frame however, would include the voluntary act of entering the car, turning
the ignition key, and driving.
(b) Omission: A person is not guilty for failing to act, even if such failure permits harm to occur to another, and even
if the person could have acted at no risk to her personal safety. **Followed by FL, MPC, and Non-MPC states**
Exception: A person has a legal duty to act in limited circumstances, if he is physically capable of doing
so. (S.C.R.A.P).
o S Statute (law enforcement, psychiatrist, lawyers, nurses, doctors, etc.)
o C Contractual dutyexpressed or implied (i.e. baby sitter, lifeguards, security guardsan
informal contract)
o R Relationship (i.e. Status relationships. Parent-child, husband-wife)
Ex. Jones v. United States, where a woman in care of two children after their mother left
them, failed to feed them. One of them died from malnutrition. She appealed her
involuntary manslaughter charge alleging that she had no duty of care to the children.
Since the existence of a duty was an element of the crime, the case was remanded for
determination. (she could be held liable for assumption of care, or contract with the
mother to care for them).
Hypo: Man and Woman lived together for many years along with her children from a prior
marriage. They lived together long enough to constitute common law marriage. Common
law marriage if behaving like a family then the husband is liable under a status
relationship for any harm caused to the children of the marriage.
o A Assumption of voluntary care or isolation from care.
If you begin the rescue and by virtue of rescuing you isolate the aid of others, then they
have assumed that care.
o P Peril (Once you create the danger you have the duty to respond and rescue those affected by
your negligence- you dont have to risk your life but do have to take steps to save others)
(c) Duty to Report or Rescue: Child Assault Good Samaritan Bill, requires witnesses of a violent or sexual
assault on a child under 14 to notify the police. (Varies depending on state statute bill). Samaritans come in at least
5 different stripes:
1. The Good Samaritan: Helps the victim by calling the police. Might even help by physically interceding
while the crime is occurring.
Page 3 of 36

2. The Hopelessly Bad Samaritan: Refuses to help. Maybe out of loyalty to the criminal, unreasonable fear
of retaliation, or callousness coupled with a perceived improbability of being identified and prosecuted
even if a duty to rescue law exists and cant be budged by this by conscience, law or punishment.
3. The Legally Swayable Samaritan: Would be bad absent a duty to rescue law, but would be swayed by
such a laws coercive or normative effect.
4. The Delayed Samaritan: Initially does nothing, maybe due to loyalty, panic, hurry, fear or uncertainty, but
later changes his mind and wants to report what he saw.
5. The Passive Samaritan: never reports the crime, but when the police come to his door looking for
witnesses he is willingbecause of remorse, fear of lying to the policeto respond questions asked about
the event.
Although the types of Samaritan might seem bad, and the Bill seems to inadvertently cause an
anti-cooperative effect, they are individually beneficial to the overall process of preventing a crime
or prosecuting a criminal for a crime.
(d) Possession: The mere possession of an object (contraband, burglary tools, drugs, etc.) would satisfy the element of
actus reus. **Followed by MPC**
Types of Possession:
o Actual possession: Something is on someones person, in his or her clothing, within his or her
physical custody, or ready reach.
o Constructive possession: The ability to exert control over the item without actual physically
having it. (legal fiction)
o Joint Possession: more than one person can possess the same item.
Elements: In order for possession to satisfy the actus reus element D must:
1. know that he has possession of the item; and
2. fails to terminate possession upon having the opportunity to do so.
Hypo: D is charged with possession of images of child porn on his computer. It is against the law to
possess images of child pornography. D says he didnt know it was on his computer. D says he got a spam
email and the link took him to a website where the images were on the home page. D claims to
immediately have closed the browser. He had no idea that his computer would cash all of the visited
websites. No possession.
o What if D knew about Cash and had the pc set to delete cash every 48hrs? He would have
constructive possession. As soon as he knew that it would be stored in cash he should have
acted towards terminating the images stored in cash. He needs to take every step possible to
delete the images and terminate possession.
(e) Vicarious Liability: Criminal responsibility for the acts of another personvery unusual todaymostly seen in
employment relationships. (Ex. State v. Beaudry: Server sold alcohol to a minor.) **Not Followed by FL**
People v. Jackson: a statute made certified pesticide applicators liable for their non-certified agents. The
Court held Defendant liable because the law made him liable for the purpose of furthering responsibility
on pesticide applications.
o If statute expressly states vicarious liability, then it is.
o In cases where the legislature is silent or ambiguous on vicarious liability, legislative intent is
looked at if it is a serious punishment, it will likely not be vicarious liability if it is merely a
fine, likely be vicarious liability.
Florida requires that statute expressly establish liability. There is not liability by implication, and if a
statute is ambiguous, there will be none. Also, it is only established by statute, not by common law.
Issue that arises Statutory InterpretationIf the statute imposes vicarious liability its not usually a
problem. If statutory language is ambiguous as to whether vicarious liability is recognized the court may
impose under the following:
o If punishment/penalty is severe the Court is less likely to construe the statute as imposing
vicarious liability.
o If punishment/penalty is minor the Court will impose the liability.
When Vicarious Liability is expressed, some courts may view it as an unconstitutional violation of due
process.
Page 4 of 36

Ex. State v. Gumingaserved alcohol to a minor. The court invalidated the statute and held that
no one can be convicted of a crime punishable by imprisonment for an act he did not commit, did
not have knowledge of, or give expressed or implied consent to the commission thereof , especially
when there are alternative means, such as civil fines or license suspension.

Mental States
(a) The prosecution must prove beyond a reasonable doubt that when D committed the act he/she had the required
mental state for each material elementmens rea has to coincide with the actus reussame time and about the
same conduct.
Material Element (MPC) Elements that cover only the matters relating to the social harm that the law
seeks to prevent or to the existence of an affirmative defense for Ds conduct. (Important elements)
Minor Elements jurisdiction, venue, federal or state. (Ex. Feola, where the SC held that the quality of
being a federal officer who was the victim of an assault was jurisdictional. No mens rea is required
toward a simple element.
D can then try and rebut with a mens rea defense such as defense of fact.
(b) Conditional Intent Satisfies Purpose
General Rule: Mens Rea can be satisfied by either an unconditional intent OR a conditional intent. (Ex. D
intends only to steal the victims car but is willing to kill him if he resists.)
(c) 2 distinct schools of thought as to Mens Rea:
Followed by Florida Specific v. General Intent Crimes (archaic and only significant today for mens rea
defenses and inchoate crimes)
o Specific Intent: D intends to commit the actus reus and something further. (Corresponds with the
MPCs purpose or knowledge)
Ex. Larceny. D intents to take and carry away the personal property of another with the
specific intent to permanently deprive another of that property.
Ex. Burglary. D commits a breaking and entering intentionally + the intent to commit a
felony inside of that structure.
Includes more than the objective fault required by merely doing the proscribed act
includes an actual subjective intent to cause the proscribed result.
Indicated by words such as knowingly, purposefully, willfully, or intentionally.
o General Intent: D has a culpable mens rea with regard to the conduct. All that is needed is Ds
intent to commit that actus reus. (Corresponds to MPCs recklessness and negligence)
Ex. Battery. D only needs to intend the offensive touching and not the injury.
Ex. Arson. D only needs to intend the burning and not the damage.
Approach is rejected by the MPC. It divides mens rea into 4 categories:
o Purpose
o Knowledge
o Recklessness
o Negligence
MPC: Splits the word intent into:
a. PurposeA person acts purposely with respect to a material element of a defense when:
1. It is their conscious object to engage in particular conduct or to cause a particular result;
and
2. They are aware of the existence of such circumstances or believes or hopes that they
exist.
Ex: D wants to kill his wife knowing that she car-pools every day. He puts a bomb in the
car. His conscious purpose is to kill his wife and hopes no one else will die. As to
everyone else in the car it was no purposeful under MPC.
Conditional Intentcan satisfy purpose. (Ex. Holloway carjacking case. D is telling
victim your money/item or your life.)
b. KnowledgeA person acts knowingly with respect to a material element of a defense when:
1. He is aware that his conduct is of a particular nature or that such circumstances exist; and
2. He is aware that it is practically certain that his conduct will cause such a result.
Ex: Husband knowingly killed the wifes colleagues, although that was not his purpose.
Page 5 of 36

Willful Blindnesssubstitute for knowledge. Occurs when a statute requires knowledge


and a D is deliberately blind to the facts. a/k/a Ostrich syndrome.
o Ex. Jewel, Case of drug smuggler that claimed not to know there was marijuana in
his car against all blatant facts.
c. RecklessnessA person acts recklessly with respect to a material element of a defense when:
1. he consciously disregards a substantial and unjustifiable risk that the material element
exists or will result from his conduct; and
o Is it substantial? Look at probability of harm and seriousness of harm. Risk of
death is always substantial no matter what the percentage.
o Is it un/justifiable? Look at Ds reason for taking the risk. (i.e. If its a life saving
medical test, or life saving medical attention on the side of the road by a doctor.)
2. his disregard of that risk was a gross deviation from what a law abiding person would do
in Ds situation.
o Ex. Pit bull case is the paradigm of recklessness. The purpose of Ds act was to
protect his marijuana and he disregarded the fact that there were kids around and
the dog was innately dangerous.
d. NegligenceA person acts negligently with respect to a material element of a defense when:
1. he should be aware of a substantial and unjustifiable risk that the material element
exists or will result from his conduct; and
o Is it substantial? Look at probability of harm and seriousness of harm. Risk of
death is always substantial no matter what the percentage.
o Is it un/justifiable? Look at Ds reason for taking the risk. (i.e. is if a life saving
medical test, or life saving medical attention on the side of the road by a doctor.)
2. his disregard of that risk was a gross deviation from what a reasonable person would do
in Ds situation.
o Ex. Pit bull case is the paradigm of recklessness. The purpose of Ds act was to
protect his marijuana and he disregarded the fact that there were kids around and
the dog was innately dangerous.
*EXAM TIP: If the fact pattern could be reckless or negligence, analyze both but to save
time just say that your analysis applies as to substantial and justifiable risk and discuss the
should be aware and reasonable person differences.
(d) Transferred Intent: If D intends to kill victim A and instead kills victim B, then D is guilty for the murder of
victim B and attempted murder of victim A because the intent transfers from the intended victim to the unintended
victim.
(e) Strict Liability: **Followed by MPC, Non-MPC, and Florida**
Common Law Rule: all crimes require a mental state.
o Exception: Strict Liability crimes where D is guilty because D committed the actus reus and no
mens rea is required. (Ex. Speeding tickets.)
Crime will also apply to Ds accomplice(s) and co-conspirators as well.
Strict liability crimes are established clearly by statutes, but the statute has to be clear, there are no
common law strict liability crimes.
In all jurisdictions Strict liability is the exception not the rule but it varies based on jurisdiction.
o MPC:
Strict liability must be expressed
Some mens rea is needed for every material element.
If statute is silent then element will be satisfied if D acted with purpose, knowledge, or
recklessness. (negligence will never satisfy an MPC statute that is silent about mens rea)
Carryover Rule: If the statute has a mens rea word about one element, it carries over to
all the other material elements of the crime unless its plain that the legislature meant
something else.
**Rule of Lenity is NOT used in MPC states**
o Non-MPC State:
If mens rea is clearly excluded then there is a Strict liability crime.
Page 6 of 36

If silent, Strict liability may apply if it is what the legislature clearly intended. This intent
can be determined by looking at: (see Balint)
Whether the statute is a public welfare or regulatory type of crime.
o If statute is malum prohibitum (crimes against the law)its possible for
it to be a strict liability.
o If statute is malum in se (inherently evil crimes)its probably not a strict
liability. (see Morrisett)
If Mens rea is unclear then Court will use legislative history and the rule of lenity.
The heavier the penalty the less likely that it will be strict liability.
Classification of Crimes
General Intent Crimes
Specific Intent Crimes
Malicious Crimes
Strict Liability Crimes
List
Battery
Attempt
Arson
Regulatory offenses
Rape
Solicitation
Malicious destruction Public welfare offenses
of property
Kidnapping
Conspiracy
Morality crimes
(statutory rape, bigamy)
False imprisonment
Larceny
Selling liquor to minors
Involuntary
Larceny by trick
manslaughter
False pretenses
Depraved-heart murder Embezzlement
Forgery
Burglary
Assault
Robbery
Intent to kill murder
Voluntary manslaughter
State of
Negligence,
Intentional, knowing,
Reckless disregard of
Irrelevant.
Mind
recklessness. Only intent purposeful, willful or
a known risk.
Required to do the act, not intent
wanton. Intent to do the
to do the crime.
crime.

Mens Rea Defenses

Not an affirmative defenseD is not admitting to all elements of the crime, rather, D is saying he has a defense
that negates the mens rea element (no intent to commit the crime).
(a) Mistake of Fact: D is not guilty of a crime if her mistake of fact negates the mens rea of the offense charged.
o Strict Liability Crimes: Mistake of fact is never a defense. (i.e. mistake of age in statutory rape cases.)
o Specific Intent Crimes: A D is not guilty of a specific intent crime if her mistake of fact negates the
specific-intent element of the offense. Ds mistake of fact must be honest, even if unreasonable.
o General Intent Crimes: A D is not guilty of a general intent crime if, as the result of her mistake of fact,
she committed the actus reus of the offense with a morally blameless state of mind. Ds mistake of fact
must be both honest AND reasonable.
o MPC Approach: A mistake of fact is a defense to a crime if the mistake negates a mental state
element required in the definition of the offense. (Doesnt distinguish between specific or general intent
crimes.) Applies to all offenses in the same manner.
Exception: The defense of mistake of fact is inapplicable if the D would be guilty of a lesser
offense had the facts been as she believed them to be. (In this case, if the mistake makes D guilty
of a lesser crime, he will be liable for that crime.)
If purpose, knowledge, or recklessness is required, the mistake needs to be honest only.
Ex: delivery guy delivering cocaine thinking it was sugar (no liability because mistake
was honest), or killing a friend in a practical joke thinking the gun was unloaded (not
purpose, because of mistake).
If negligent is required, both honest AND reasonable
o Sexually Battery Crimes: Jurisdictions today do not allow a mistake of fact defense regarding the
complainants consent (considered a strict liability element). **Followed by Florida**
Page 7 of 36

Statutory Rape: Jurisdictions today do not allow a mistake of fact defense regarding the complainants
consent (considered a strict liability element). **Followed by Florida**
Some states are changing this however, where D has done everything possible to avoid having sex
with an under aged person.
(b) Mistake of Law: D tries to negate the mens rea element of a crime by arguing that (1) he didnt know that there
was such a law, or (2) he didnt know that law existed but didnt know it applied to his conduct.
o General Rule: Ignorance of the law is no excuse.
Exceptions: Applicable only to specific intent crimes.
If D lacks fair notice of a legal duty imposed by law (usually occurs where law isnt
published yetdue process issue)
D reasonably relies on an official statement of the law that turns out to be wrong.
o An appellate decision that is later overrules
o A statute that is later declared void or unconstitutional
o An official interpretation of the law by someone with authority
Ex: Cox v. Louisiana, D was charged for protesting near a courthouse,
despite their being instructed to do so by a sheriff. Ct held that the sheriff
had responsibility for clarifying terms of the law delegated to him and
other local officials. D here could rely on the sheriffs adviceif bad then
he had a defense.
Ex: Hopkins v. State, where D (a reverend) erected signs advertising his
marriage services relying on the advice of a state prosecutor. Ct held it
was no defense.
When there is a mistake of law about non-criminal law (i.e. property law)
o Ex: Smith-David, where D destroyed some fixtures that he installed while he was
renting a property. Ct held that because he thought it was his own property, the
principle that ignorance of the law is no excuse only applies in criminal law, not
property law.
When law specifically requires knowledge that Ds conduct is criminal.
o Ex: Baker, D knew he was selling counterfeit Rolexes, but didnt know it was
illegal. Ct held that knowledge for this crime was not required thus no defense
because conduct didnt require him to know it was illegal.
o Ex: Ratzlaf v. US, where D was charged with structuring transaction to evade
banks obligation to report transactions over $10K. Ct held that mistake of law
was a defense because although D knew that he was trying to evade the reporting
requirements, he didnt know that it was illegal to do so (fed statute required such
knowledge).
o MPC Approach: Follows the common law rule that mistake of law is not a defense.
The defense of mistake of law is inapplicable if the D would be guilty of another crime had
circumstances been as he believed them to be.
Exceptions:
If it negates mens rea when D reasonably relies on an official statement of the law that
turns out to be wrong
D lacks fair notice of a legal duty imposed by law (usually occurs where law isnt
published yetdue process issue)
(c) Intoxication: This is a mental state defense due to physical or mental disturbance caused by introducing alcohol or
intoxicating substances into the body.
o Voluntary Intoxication: Self-inducedknowing you take drugs and knowing they can have a certain
effect.
Applies to specific intent crime in some states (Not general intent or strict liability crimes)
Could possibly apply in an actus reus situation if the actus reus is an alibi defense. [i.e. D argues
that he was so drunk that he was passed out & unconscious. So someone else committed the
conduct. This is an alibi (it wasn't me because....)]
Florida, does not allow this defense.
o Involuntary: Not self-induced. Can be a defense in some states under the following circumstances:
Coercion: D is drugged by a 3rd party; or (**Followed by Florida**)
o

Page 8 of 36

Mistake: D is mistaken about what he is ingesting


Pharmacy gives him the wrong medication
Prescribed Medication: D becomes unexpectedly intoxicated from ingestion of a medically
prescribed drug, perhaps due to an allergic reaction.
Florida, allows this defense to negate specific intent when the use of a controlled substance,
issued to D by a medical practitioner, and D properly used that medication.
MPC Approach: (No distinction between specific or general intent crimes)
Any form of Intoxication is a defense if it negates an element of the crime. (i.e. Voluntary,
Involuntary, Pathological)
Exceptions:
o Recklessness CrimesIf D is charged with an offense for which recklessness
suffices to convict, he cannot avoid conviction by proving that, because of
intoxication, he was unaware of the riskiness of his conduct.
Pathological Intoxication: A rare reaction to a voluntary ingestion of small amounts of
intoxication. (Similar to FL that it is a voluntary ingestion but an unexpected reaction.
Temporary Insanity: Involuntary and Pathological Intoxication can be an affirmative defense,
if it causes the D to lack the substantial capacity to appreciate the criminality of his conduct or to
conform his conduct to the requirement of the law. (Meaning, intoxication might be a form of
insanity under certain circumstanceshe didnt know right from wrong, or didnt understand what
he was doing.)

Causation

When a crime requires a result, the prosecution must prove not only that the result occurred but also that Ds
conduct caused that result. (Crimes without a result, i.e. theft, reckless driving, attempt, dont require proof of
causation.)
Causation is needed for result crimes:
o Murder
o Crimes that damage property
o Crimes that cause bodily injury
The State has to prove 2 elements for causation: (conjunctive)
1. Cause-In-fact (actual cause)A person is not guilty of an offense unless she is an actual cause of the
ensuing harm. Conduct is the actual cause if the result would not have occurred but for the actors conduct.
(used to identify potential candidates for liability) **Followed by Common law and MPC**
But-for Test: But for Ds act would this result have occurred? If no, then D is the cause in fact.
Modified Test: But for Ds act would this result have occurred when it did? [test only applied
when accelerating inevitable result cases]
Substantial Factor Test: Whether the act was a substantial factor in procuring the result? [test
only applied when concurrent sufficient causes] (i.e. 2 assassins that kill at the same time, or hypo
of couple that battered the baby.) **Not Followed by MPC**
Hypo: Jeffrey and Jennifer beat Jeffreys 6-year-old child, one in the night, the other next
morning, he died in the afternoon from a blow, but the doctor is not able to determine who
could have strike it. Both ask to be acquitted for failure to prove causation. There is no
test for this. It is difficult to make out causation given the circumstances. Prosecutor will
usually go for a lesser charge.
2. Proximate Cause (Legal cause)A person who is an actual cause is not responsible for it unless she is
also the proximate cause. (used to determine who among the candidates are accountable for the harm.)
Issue arises when talking about possible intervening causes. The Q then is whether the event can break the
chain of causation and possibly exonerate the person from liability. (TIP: If person shoots someone with a
gun, proximate cause need not be discussed.)
The result must occur in a natural and continuous sequence, though causation is not limited to
the immediate or most obvious result. The D is responsible for the natural and foreseeable
consequences that follow, in the ordinary course of events, from the act.
Natural and foreseeable consequences (Predicated upon foreseeability. i.e. A doctors
negligence is foreseeable)
Page 9 of 36

The proximate cause of an injury is that which, in a natural and continuous sequence,
unbroken by any efficient intervening cause, produces an injury, and without which the
injury would not have occurred.
Even if D is the cause in fact of someones death, there is no liability unless there is a proximate
cause MUST be the direct cause and reasonably foreseeable.
If intervening cause is dependent of the initial act, no causal chain is broken, and there is
proximate cause, which is foreseeable. But when independent and unforeseeable, the chain is
broken.
Superseding gross medical negligence or anything unexpected (bolt of lightning) always
breaks chain of causation.
Responsive means that this interruption does not break chain of causation because it is a
response to Ds conduct. If wound is inevitably fatal, then Ds act would be enough to
bring about the victims death. If would is not fatal, then a hospital procedure can possibly
break the chain of causation.
Hypo: D stabs E and sends him to the hospital. After being treated and while leaving the
hospital, E gets struck by lightening and dies. Is D liable for the death? No. The lightening
striking was an intervening superseding event cutting off liability. Its not a nexus to the
stabbing nor foreseeable.
Something you expect
Ex: State v. Echols, where D robbed the victim, the victim when after him, fall and broke
her hip. D was found liable because going after him and falling was a natural consequence
of the robbery. Even if victim had overreacted, he still would be found liable, because his
act was a concurrent proximate cause.
Ex: People v. Kibbe, where D left the victim in the middle of a road, drunk, without
glasses and his pants off in the snow, when the victim when to the road to get help, he was
killed by a car. D was found liable because death was foreseeable since they put him in a
position where he could not extricate himself, and because victims actions to get help
were foreseeable too.
Ex: Commonwealth v. Root, where D was racing with victim, victim in an attempt to
pass him swerved to the opposite lane of the road, collided with a truck and died. Court
found D not guilty because the independent and unforeseeable action of the victim broke
the chain of causation. Victims suicidal action was an intervening cause.
o Other courts have considered that the victims actions are foreseeable, as they are
a direct consequence of the racing. Now, if victim would be distracted by a call,
the chain would have been broken.
Can be a reaction to medicationvictim ends up in hospital & has a reaction to meds even
rare reaction is foreseeable.
Ex: State v. Jenkins, where D stabbed the victim, once in the hospital, died from a
reaction to the dye. D was found guilty for the death, because he would not have needed
that test had D not stabbed him. The court held that a D takes his victim as he finds him
(eggshell rule). One who inflicts an injury on another is deemed by law to be guilty of
homicide where the injury contributes mediately or immediately to the death of the victim.
Simple medical negligence is always foreseeable (gross would be unforeseeable)
Ex.: People v. Flenon, where D shot at the victim, they have to amputate a leg, and make
a transfusion. Victim died from hepatitis acquired in the transfusion. He was found liable
because medical negligence was foreseeable. Only gross negligence would be
unforeseeable, and eggshell rule applied. (Pre-existing condition does not break chain of
causation.)
Intervening causes (can break causation link): If a 2nd person is involved in bringing about a
result required for criminal liability, the 2nd actor becomes an intervening cause that may break
the causal chain between the 1st actor and the result.
The mental state and blameworthiness of the second actor play a major role in determining
whether or not the second actor will relieve a first actor of causal responsibility.
Hypo: A stabs B in the leg. Along then comes the Doctor and chokes B to death. The
doctors act would be the intervening superseding act.
Page 10 of 36

Persons religious beliefs and refusal to undergo treatment do not break the chain of causation
(eggshell).
If a second person is involved in bringing about a result, the mental state and blameworthiness of
the second actor plays a major role in determining whether or not the second actor will relieve a
first actor of causal responsibility. Ex: courier delivering the bomb unknowingly.
Year and a Day Rule [abolished]: If D committed an act against a victim and the victim dies a
year and a day after, D is no longer liable. (some states, although abolished, have enacted
something similar.)

Homicide

Definition of Homicide (General) The killing of one human being by another human being.
o Innocent killings are not criminal thus not punishable.
o Criminal homicide is an act that takes place without justification or excuse.
o Human being definitions: questions of when life begins and ends.
In Florida, human being is when life begins. They have a separate statute on fetal homicide.
Tests Applied: (**FL uses both when applicable**)
Breathing Test (Traditional)Life ends when the heart/breathing stops.
Brain Dead (Modern)A human being dies when the brain is dead. (if person is on life
support)
by another human being was added because without it, an attempted self murder (suicide)
would be charged as murder under its traditional definition.

Requirements for Homicide: A.M.D.C.


o Conduct (Actus Reus)Either an affirmative act or a failure to act where there was a duty to do so.
o Mental State (Mens Rea)State of mind of D concurrent with the act that is also used to determine the
degree of the punishment.
o Death (Corpus Delecti)Resulting death from the act.
o CausationCausal link between conduct and result. (actual or proximate)

Classification of Homicides under Common Law:


o Murder: an unlawful killing of a human being by another human being with malice aforethought.
Actus Reus: At common law, mainly dealt with Qs of the beginning and ending of life.
EXAM TIP: Occasionally there may be an issue about the beginning and ending of life,
or voluntariness of an act but mainly the issues will be on mens rea. Causation is always
an element at common law and modern statutory law even for felony murder but it comes
in through a limitation on the felony murder rule.)
Mens Rea: Malice Aforethought. Occurring in 4 ways:
Intent to kill
Intent to commit serious bodily harm but death occurs
Depraved heart
Felony murder
o Manslaughter: Everything else.
o * The common law used recklessness for involuntary manslaughter, which is still the standard today
(even when Florida calls it "culpable negligence").

Classification of Homicide under Non-MPC (modern statutory law):


o Murder
1st Degree: A killing committed in one of 4 ways
Brutal or Heinous ways (poison or ambush) **Not Followed by Florida**
Premeditated and Deliberate
Felony murderinvolving an enumerated dangerous felony or its attempt.
2nd Degree: all other murders
Reckless murder w/ extremely depraved indifference.
Page 11 of 36

Felony murder during non-enumerated felonies.


Manslaughter
Voluntary
Imperfect Self-Defensewhere D satisfies all elements of self-defense except for one.
(i.e. D had an honest but unreasonable belief that deadly force was necessary.) **Not
Followed by Florida**
Provocation (Heat of Passion)
Involuntary
Involves Lawful Actsan unintentional killing caused by a lawful but reckless act.
o Ex. People v. Hall, where D, an experienced skier, killed another skier while
skiing at a high speed and out of control. The court found him liable because his
act involved a substantial risk of death or serious injury, the risk was unjustifiable
(recreation), he was aware of the risk (experienced skier), and he consciously
disregarded the risk. His conduct was not the conduct a reasonable skier with his
experience.
Involves Unlawful Acts(Misdemeanor Manslaughter)an unintentional killing
during the commission of an unlawful act.
o This is the Misdemeanor counterpart of felony murder.
o **Not Followed by Florida or MPC and is becoming disfavored in most
states**
Classification of Homicide under Florida:
o Murder:
1st Degree:
Premeditated murder
Felony murderwhen D or accomplice kills during an enumerated dangerous felony.
Death due to certain drug transactionsspecial form of felony murder but laid out
separately in the 1st degree felony murder statute.
**EXAM TIP: If Q says did D commit 1st degree murder in FL, ask yourself if its any of the 3
above**
2nd Degree:
Reckless Murder w/ Depraved Indifference to Life
Felony Murderwhere a non-felon kills during Ds enumerated dangerous felony.
3rd Degree:
Felony murderwhere D or accomplice kills during a non-enumerated felony.
o Manslaughter:
Manslaughter by Act (Corresponds to Voluntary Manslaughter)
Culpable Negligence (Corresponds to Involuntary Manslaughter)
Manslaughter by ProcurementD persuades, induces or encourages another person to commit
an act and the other persons act results in the unintentional death.
Classification of Homicide under MPC:
o Murder:
Purpose/Knowledge
Recklessness
o Manslaughter
Reckless Act
Extreme Mental or Emotional Disturbance
o Negligent Homicide
A homicide committed negligently. D engaged in highly risky conduct, and should be aware, but is
not aware, of the risk of death.
**Not Followed by Florida or Non-MPC**
o

Purposeful/Intentional Killings Mens Rea: Purpose or Knowledge (Issue: Whether intent to kill is formed
from Premeditated & Deliberate OR under emotional circumstances.)
o Non-MPC (Traditional State):
Page 12 of 36

If Premeditated & Deliberate Intent = 1st DEGREE MURDER


If Intent formed in Emotional Circumstances = VOLUNTARY MANSLAUGHTER (Test
applied Provocation test)
Florida:
If Premeditated & Deliberate Intent = 1st DEGREE MURDER
If Intent formed in Emotional Circumstances = MANSLAUGHTER BY ACT (Test applied
Provocation test)
MPC:
If Premeditated & Deliberate Intent = MURDER
If Intent formed in Emotional Circumstances = MANSLAUGHTER (Test applied Extreme
Mental or Emotional Disturbance Test)

Premeditation and Deliberation: Mental process of thinking over beforehand for a period of time, however short
after which intent to kill is formed.
o To Premeditate is to think about beforehand.
o Rule: Premeditation requires some advanced planning, but it need not be long range planningit could
arrive instantly. **Followed by Florida**
Ex: State v. Bingham: D raped and strangled a mentally retarted girl while on the way back home.
D argued that court erred in determining that because strangulation takes 3-5min the act was
premeditated and deliberate. Ct held that the state needed to show not only that (1) there was
enough TIME to deliberate but also (2) proof that D ACTUALLY deliberated. Usually the
evidence used to show proof is the weapon(s), motive, and planning related to the killing.
Strangulation is not premeditation per se.
o To Deliberate some sort of consideration or weighing the act with a cool head.
Ex: Commonwealth v. Carrol: Husband that shot wife in her sleep for nagging. Not having a plan
after killing someone is not necessary and doesnt mean that the murder in itself wasnt planned
(deliberated). The use of a deadly weapon, in a vital part with the use of 2 shots, created an
inference of deliberation.
Provocation (Heat of Passion): An intentional killing created in the heat of passion resulting from a reasonable
provocation.
o Common Law (Florida Uses): (Requirements)
(1) Legally adequate provocation (objective standardwords alone are never enough)
Categories:
o Severe battery
Ex: State v. Lawton, where D was beaten in a bar, and went back 10
minutes later to kill one of the beaters, and court found 10 minutes was
not enough time to cool off.
Words alone are never enough.
o Vicious/Violent attack on close family member
o Observation of spousal adultery
Elements:
1. Sexual intercourse. Any other indication of adultery does not qualify.
2. Marriage. Non-marital relationships do not qualify.
3. Direct observation of sexual intercourse. Knowledge/hearing of
adultery alone is not enough.
Ex: Dennis v. State, D killed wifes cocaine addicted lover when he
caught them in a sexually intimate situation. D argued that he was
provoked by the sexual position he found them in plus the fact that Victim
has smoked crack in front of his child. Ct held that for the provocation to
be valid, sexual intercourse was needed.
Also, D could not act
based on a combination of events leading up to the provocation that
caused the actthis would not be an act in the heat of passion.

Page 13 of 36

Last Straw Doctrine: Where D accumulates lesser forms of


provocation and then claims that the most recent was the last
straw that pushed him over the edge. *NOT ACCEPTED*
o Mutual combat or illegal arrest
(2) The provocation must cause D to become impassioned.
Causation Provocation must be the cause of impassion.
D must actually be impassioned.
(3) No reasonable time to cool off between provocation and act.
Did D have enough time to cool off?
Did D actually cool off?
o Ex: State v. Pierce, the court refused to determine how much time would be
reasonable, but said that the next day was too much time.
o Presents a problem in Battered Spouse cases.
(4) D must not have actually cooled off before the act.
Long Smoldering Passions Doctrine (Minority/Rare): D appears to cool off but
provocation is burning under the surface of his emotions and then erupts.
Rekindling Doctrine (Minority/Rare): D has cooled off but a reminder sends him back
into the moment and into a rage. **Not Followed by Florida. Rarely used in MPC**
Florida Approach: (Same rule as Common law)
(1) Mitigation of the Charge: If D commits an intentional killing in the heat of passion; the homicide
charge will be mitigated to a lower degree of murder or to manslaughter by act.
(2) Complete Acquittal: (Statutory ProvisionExcusable Homicide) If D commits an accidental
killing in the heat of passion then the killing is excusable and D is acquitted entirely.
MPC Approach: Punishment is according to mens rea. Instead of provocation, where there is an
intentional killing, the murder charge will be mitigated where there is:
(1) Extreme mental or emotional disturbance; and
(2) Reasonable explanation or excuse for it.
Reasonableness is determined from the perspective of someone in Ds position, under the
circumstances, as he believes them to be. (Subjective)
Only mitigates murder charges, not manslaughter or negligence homicide.
Idiosyncratic moral values wont work (racism, homophobia), but handicaps can see things
differently, and that is valid.
Battered Woman Syndrome: (See Brooks v. State)
Traditional View: All provocation test requirements must be met.
Modern View: The following collectively form provocation:
Past abuse;
Fear of future abuse; and
The present threat.

Reckless Killings Mens Rea: Recklessness (Issue: Whether D was reckless AND had a depraved
indifference to life) **Must analyze both Recklessness and Depraved Indifference**
o Non-MPC (Traditional State):
If Recklessness with Depraved Indifference to Life = 2nd DEGREE MURDER
If Recklessness without Depraved Indifference to Life = INVOLUNTARY MANSLAUGHTER
o Florida:
If Recklessness with Depraved Indifference to Life = 2nd DEGREE MURDER
If Recklessness without Depraved Indifference to Life = MANSLAUGHTER BY CULPABLE
NEGLIGENCE
o MPC:
If Recklessness with Depraved Indifference to Life = MURDER
Recklessness is presumed and rebuttable if actor engages or is an accomplice in the
commission of, or attempt to commit, or flight after committing or attempting to commit
robbery, rape, or deviate sexual intercourse by force or threat, arson, burglary, kidnapping,
or felony escape. (equivalent to felony murder)
Page 14 of 36

If Recklessness without Depraved Indifference to Life = MANSLAUGHTER

Depraved Indifference: (Recklessness on Steroids)


o Ex: Barry, where D, who owned a fighting pit bull, kept his fence open and a 2yr old was killed by dog.
The Court held that there was sufficient circumstantial evidence proving Ds recklessness with depraved
indifference to life. He knew that the dog was a fighting dog (although had never attacked a human
before), was trained for that purpose, knew that there were kids next door; and ignored it in order to protect
his illegal activity (marijuana plants).
o Ex: Fleming, D drove intoxicated and out of control and killed a woman. D had a DUI record and was
aware of the risks of driving as he did. The court held that malice required for murder was implied by the
inherently dangerous conduct with depraved indifference to human life in which D engaged, regardless of
purpose to kill. Risk was substantial because it could cause death.
The court in these cases can time frame the decision to drive at an earlier point when D was sober
and made the decision of putting the key in the ignition and go drinking while driving.
o Ex: Case of the woman that while driving drunk hit a pedestrian, and after she failed to extricate him from
her windshield, she failed to call the authorities, took him home, and let him die in the windshield. Her
actus reus was her failure to rescue him after putting him in peril, and her mens rea was recklessness. She
had a depraved indifference to human life.
Recklessness w/o Indifference: An unintentional killing caused by a lawful act that was committed recklessly.

Negligent Killings Mens Rea: Negligence (Issue: Whether an unintentional killing was the result of Ds
negligence)
o Non-MPC: Not Applicable
o Florida: Not applicable
o MPC: Negligent Homicide (Less serious than manslaughter)
Ex: State v. Williams, parents did not provide medical care to their son because they were afraid
that the state would take their child away. The child died and the court found them liable for
negligent homicide, when they had a natural duty of care to the child, they breached that duty by
failing to provide medical care, and the death was caused by such failure. However, the court did
not require gross negligence, because the statute only required ordinary negligence.

Felony Murder:
o Common law Rule: If death occurred from conduct during the commission or attempted commission of
any predicate felony it was murder. No mens rea was required regarding the death. (Mens rea was
borrowed from the underlying predicate felony)
o Florida Felony-Murder (782.04): (Enumerates for both 1st and 2nd degree felony murder)
1st degree: D kills or causes the death of another person during the commission or attempted
commission of an enumerated felony. (agents)
Killing does not need to be in furtherance of the felony
2nd degree: Someone who is NOT one of the co-felons causes the death during the commission of
an enumerated felony, (i.e. a victim, bystander, police officer).
Non-agents, but still felony murder
Need not be an unlawful death, because D created the situation.
Death can even be by suicide
3rd degree: D killed or caused to be killed while committing or attempting to commit an
unenumerated felony. (agents)
Killing does not need to be in furtherance of the felony.
o MPC (and few states): No felony-murder rule at all. The typical felony-murder situation would be
reckless murder with extreme depraved indifference to life.
Rule: If D or accomplice is engaged in an enumerated felony, its attempt, or flight therefrom, then
there is a rebuttable presumption of recklessness and extreme indifference to life. (versus the other
felony murder where the D has no chance to rebut anything because no mens rea is required.)
MPC also recognizes that the predicate felony must be independent from homicide.
(Merger limitation)
Page 15 of 36

The burden shifts to the D to rebut the charge otherwise it will automatically be presumed
and they would be charged with Reckless Murder.
Limitations on Felony Murder: Because Felony murder is so vague they developed these limitations to
determine when it does apply. **EXAM TIP: If after going through the limitations you are coming to
the conclusion that it cant be felony murder at all, dont conclude innocence. Consider any other
possible charges such as recklessness w/ depraved indifference or involuntary manslaughter**
(F.R.E.I.M)
In Furtherance: (Proximate causation) Killing must be in furtherance of an underlying felony
there has to be a causal connection between the predicate and the death.
Ex: Weick v. State, D entered a house to steal marijuana, the owner of the house shot at a
co-felon, and D was charged with felony murder. Ct dismissed the charge because in that
jurisdiction the killing has to be in furtherance of the predicate felony. (So if police officer
or victim kills a felon it is not considered to be in furtherance of the felony but rather in
opposition to the felony).
For states that follow this limit, a killing by a non-felon does not trigger felony-murder
because it is in opposition to the predicate felony.
As to whether a state follows this limitation, Oimen case tells us that this depends on the
wording of the felony murder statute. Florida covers this by holding the felon liable for
2nd degree murder if killer is a non-felon in the case of enumerated felonies.
Agency: A felon is only responsible for homicides committed in furtherance of the felony, by a
person acting as the felons agent, otherwise it is not considered felony-murder.
Florida covers this limitation by 1st and 3rd degree felony-murder, both requiring an act
by a co-felon.
For states that follow this limit, ask yourself who did the killing. If the actor is an
accomplice, then all co-felons are guilty. If its anyone else then it is not a felony-murder.
For all other states that dont use this limit, anything goes. Actor is liable for victims death
as long as it is connected to the predicate.
Res Gestae: Death has to be connected to the predicate felony by time, place or causal connection.
As part as one continuous transaction. Issues often arise with flight from the scene.
Applies to both enumerated and unenumerated felonies.
Ex: State v. Adams, Ds ran from Marshals after robbing a gas station and while fleeing
one of the accomplice shoots and kills a Marshal. Ct held that res gestae of the crime
continuous during flight from the crime and, in majority of states, continuous right up
until the Ds reach a place of apparent safety. Any death after that point cant trigger felonymurder. **Followed by Florida**
D can argue that his act did not cause the death. (Causation argument)
Majority Rule: The mere fact that a death occurs, in a temporal sense, during the
commission of a felony, is insufficient to trigger the felony-murder rule. There must also
be a causal nexus between the felony and the death. However, a death that occurs after the
felony committed, but during the escape from the site of the crime, falls within the res
gestae of the felony.
Enumeration: States limit felony murder to enumerated felonies. (Most deadly and dangerous)
(B.A.R.R.K.E.)
Burglary
Arson
Robbery
Rape
Kidnapping
Escape from lawful custody (i.e. from jail)
Inherently Dangerous: A felony is inherently dangerous when it carries a high probability of
death. This is the test used by jurisdictions that do not enumerate felonies, and limit the crime to
homicides committed which occur during inherently dangerous felony only. Either of two tests can
be used: **Not Followed by Florida** **EXAM TIP: If the test gives you a fictitious state
Page 16 of 36

then always apply both tests and state that depending on the state this is what would
happen**
In the abstract:
o Inherently dangerous every time that it is committed. (look at the elements of the
crime)
o If there is a safe way of committing the crime, then its not inherently dangerous.
Ex: State v. Sanchez, driver was speeding with passengers in the car.
Cops chased him and against the will of the passengers, he took a 90degree turn at high velocity causing the death of one. The Court held that
D could have eluded officers in a safe way but willfully chose to be
reckless.
As applied:
o Looks to Ds conduct in that specific factual circumstance.
Merger: The predicate felony must be independent from homicide.
Applies only to unenumerated felonies.
Ex. Barnett, D killed brothers ex-girl friends boyfriend with a hoe. Ct held that assault
w/ deadly weapon is not independent from homicide so it cant be used to support a
felony-murder prosecution.
Ex. Sturdivant, D killed a 2yr old of a slap that made him fall on the floor. Ct held that
aggravated child abuse that killed didnt merge because it was enumerated.

Misdemeanor Manslaughter (equivalent to felony-murder) [MINORITY]


o Rule: Death occurring as a result of the commission or attempted commission of a misdemeanor.
Ex: US v. Walker, where D carried a gun without a permit. The gun fell, triggering a shot, and
killing someone.
o Limitations apply here as well but most states are not applying this anymore.
o **Not Followed by Florida or MPC and is becoming disfavored in most states**
o **EXAM TIP: Be aware for this type of homicide on the exam in a non-MPC state**

Justification Defenses (Affirmative Defenses) S.S.N.O.C


(a) Affirmative Defense: D admits to all elements of the crime, but it not held criminally liable.
(b) Justification Defenses: D admits to all elements of the crime, but argues that there is no wrongdoing because the
acts were justified. (**defenses extends to accomplices**)
a. Self Defense: D is a non-aggressor who reasonably believes that force is necessary to protect himself from
the imminent use of unlawful force by another. Force must be proportionate to the harm threatened. (It is
never okay to repel force using deadly force unless necessary)
i. Elements:
1. D has to be a non-aggressor
a. Rule: Aggressors dont have the right to use self-defense justification.
i. Exception: They can regain the defense in circumstances, if D withdraws
from the conflict and communicates that withdraw clearly to other but the
other continues to attack.
2. Reasonably believes that such force is necessary to protect himself or a 3rd party
a. It need not be an accurate beliefmust only be honest and reasonable.
b. Imperfect Self DefenseA situation where almost all the elements are satisfied
with the exception of one. D has an honest believe that deadly force is necessary
but is an unreasonably belief such force is necessary (will mitigate charge to
manslaughter) (Some States) **Not followed by Florida**
3. from imminent use of unlawful force by another
a. unlawful means not using force in the exercise of a legal right (i.e. police officer)
b. Imminence means impending.
i. Ex: State v. Norman, where D killed husband of 20 plus years while he
was asleep, due to his extreme physical, mental, and emotional abuse. Ct
held that defense was not justified because the danger was not imminent
Page 17 of 36

as defined by the statute. Although D argued that it would have been upon
the husbands awakening.
4. Force must be necessary and proportionate to the harm threatened.
a. Common law duty of retreat: Deadly force is not necessary if D knows he can
retreat in complete safety (subjective). (It must be an absolute knowledge, not a
should have known). Once D has retreated as far as possible, then deadly force
can be used as a defense.
i. Exception: Castle Doctrine D is not required to retreat at home even if
possible to do so safely.
b. Stand your ground Doctrine: (Extends the castle doctrine and provides D with
immunity from criminal prosecution and civil suit.Florida) Sect 776.012
i. D may use deadly force and has no duty to retreat in 3 circumstances:
1. D reasonably believes that deadly force is necessary to prevent
imminent death or great bodily harm to himself or another;
2. D reasonably believes that deadly force is necessary to prevent
the imminent commission of a forcible felony (crime of violence)
3. Under circumstances permitted, under Sect 776.03, which in turn
expands the castle doctrine to include a dwelling residence and
occupied vehicle. (Consult statute for definition of these)
ii. If any of the above occur, D is presumed to have a reasonable fear and
thus can use this defense. Presumption does not apply in 4 situations:
1. If D is engaged in unlawful activity;
2. If victim has the right to be there;
3. If victim has the right to remove the person from that place; or
4. If victim is a law enforcement officer in the execution of official
duties and D knows this.
**D can still use the defense in these situations but doesnt
benefit from the presumption.
iii. If D is not engaged in an unlawful activity and is attacked in any place
that he/she is allowed to be, then D has no duty to retreat. Sect 776.031
1. Even if D is disqualified from using this provision, for instance
unlawfully possessing a gun, D is still eligible to argue the Stand
your ground immunity under one of the other statutes.
iv. If D is prevented from using this immunity, D can still argue self-defense
justification.
ii. MPC Approach: Recognizes self-defense but not stand your ground.
1. Non-deadly force can be used if D believes that it is immediately necessary on the present
occasion.
2. Deadly force can be used if D believes it is necessary to protect against death, serious
bodily injury, death, or rape:
a. Exceptions:
i. If D provoked the use of force against himself in the same encounter
ii. D can avoid force by surrendering possession of a thing, to someone
asserting a claim of right.
iii. D can not use deadly force if he can avoid force by complying with the
demand to abstain from any action that D has no duty to take;
iv. If D knows of a safe avenue of retreat.
1. Retreat, under MPC, is required everywhere except at home or in
the work place.
a. Limited in the work placeD must retreat at work if he is
the first aggressor, or is attacked by a co-worker at work.
i. If D is attacked at work by someone that doesnt
work there, then D has no duty to retreat.
b. Defense of Others: Identical to Floridas Self-defense rule.
i. Majority Rule: D must have a reasonable belief that the other party has the right to use selfdefense.
ii. Minority Rule: D must have a correct belief that the other party has the right to use self-defense.
Page 18 of 36

iii. MPC:
1. If crime involves mens rea of Purpose or Knowledge, then D must honestly believe that
force is necessary to protect the 3rd party.
2. If crime involves mens rea of Negligence or recklessness, the D is responsible if he is
wrong about the need to use force.
c. Necessity:
i. Common law:
1. D faces a clear and imminent danger
2. Danger must come from natural forces
3. Direct causal relationship between Ds crime and harm avoided. (D has to use measures
appropriate to the situation)
4. D had no apparent legal alternatives
5. D chose the lesser evil
6. D didnt create the dangerous situation (clean hands)
a. Ex: Dudley, where Ds were stranded at sea and ate the youngest kid claiming
necessity. Ct held that necessity cant be used in homicide cases.
ii. Florida: Follows common law
1. Medical Necessity Defense: (a fact specific application of the common law necessity
defense) Recognized under limited circumstances
a. Ex.: Husband and wife who contracted AIDS and after being unable to obtain a
prescription, decided to grow marijuana.
iii. MPC: (Choice of evils defense)
1. Harm need not be imminent
a. The harm or evil sought to be avoided by such conduct is greater than that sought
to be prevented by the law defining the offense charged.
2. The actor believes to be necessary to avoid a harm or evil to himself or another (D
doesnt have to have a reasonable belief)
3. Threat not limited to natural forces (includes human forces)
4. CAN be used in homicide
5. No clean hands requirement.
a. Exception: If D is charged with crime of recklessness or negligence, the D
couldnt have been reckless or negligent in creating the danger or evaluating the
necessity.
iv. Civil Disobedience (Type of necessity): Can be
1. DirectD must be violating the same law he is protesting in order for the defense of
necessity to apply. (Law being protested must be unconstitutional not constitutional right)
2. Indirect (Necessity is never a defense)
a. Ex: Schoon, where Ds protested in an IRS office by throwing simulated blood
everywhere because of Americas involvement in El Salvador. Ct held that the act
didnt violate the same law that they were protesting against.

Excuse Defenses (Affirmative Defenses) D.I.A.


(a) D admits to all elements of the crime but argues that the wrongdoing is excused. (Personal to D and not applicable
to accomplices.)
(b) Duress: This defense serves to excuse behavior where extrinsic circumstances compel a person to perform
unlawful acts, which he did not otherwise wish to do.
a. Common law Elements:
Immediate threat of imminent death or serious bodily injury to D or Ds family (not nature)
Threat comes from a human being;
Ds criminal act was a reasonable response; and
Defendant didnt create the situation (clean hands).
o Not applicable to homicide; and
o MPC eliminates this requirement absolutely, and does not limit the defense to a family
member, can be a stranger
b. Florida: Treats Duress and Necessity the same. (Not Statutorycommon law)
c. MPC:
Page 19 of 36

D must face a threat of unlawful force;


Threat can be against any person (not limited to D and Ds family)
No imminence requirement
Person of reasonable firmness in Ds situation would yield to the threat
D didnt recklessly, or negligently put himself in the situation
Applies to all crimes including homicide.
d. Ex: Contento-Pachon, D was threatened to smuggle cocaine in his stomach from Colombia to the US
otherwise his wife and kid would be killed. Ct held that threat was imminent because if D refused his
family would have been killed.
(c) Insanity: Recognized under 3 situations
a. Incompetence to stand trial: Relates to the defendants ability to understand the criminal proceedings.
While insanity can extinguish liability, lack of competence bars prosecution while the defendant remains
incompetent. (D would be institutionalized in the mean time)
b. Insanity: Relates the defendants mental condition at the time of the offense. Sanity is presumed so D has
the burden of proofFlorida has a clear and convincing evidence standard (very high).
MNaghten Test: [Majority Rule and Florida] D is legally insane if, at the time of the act, D
was laboring under such a defect of reason, arising from a disease of the mind, and didnt know
either:
o the nature and quality of his act; or
o that his acts were wrong.
Defect of reason the ability to reason was impaired but had to be because of a
disease of the mind.
Disease of the mind could be organic deterioration (Bran function i.e.
Alzheimer, traumatic brain disorder). Functional deterioration (Mind function
i.e. Schizophrenic)
Florida, adds 2nd prong D didnt know what he was doing or its consequences,
if he did, then D didnt know what he was doing was wrong.
Wrongmeans legally, prohibited, morally wrong by society (states
difference on this, no majority view)
Ex: Crenshaw, Ct held that in those circumstances, wrong meant legally
wrong.
Deific Decree Exception: D actually believes that a supreme being ordered his
act. (Insanity is rarethis is even more rare)
D does not fit within McNaghten but is made as an exception in those
situations. (Applies in PTSD and Post Partum Psychosis situations **BE
ALERT FOR THOSE**)
MPC Test (Substantial Capacity Test): D is not responsible for her criminal conduct if, at the
time of the conduct, as the result of a mental disease or defect, she lacked substantial capacity to:
o Appreciate the criminality of her conduct; or
or, in the alternative, at the option of the legislature adopting the Code, the moral
wrongfulness.
o to conform her conduct to the requirements of the law. (i.e. Irresistible Impulse test)
N/a to anti-social conductmust be a mental defect/disease.
Under MPC, intoxication may satisfy the substantial capacity test.
Irresistible Impulse: (a/k/a policeman at the shoulder test) D is insane if, at the time of the
offense (some states have added this as a 3rd prong to the MNaghten test)
o D acted from an irresistible and uncontrollable impulse;
o D lost the power to choose between right and wrong, and to avoid doing the act in
question, as that his free agency was at the time destroyed; or
o Ds will has been otherwise than voluntarily so completely destroyed that his actions are
not subject to it, but are beyond his control.
Durham Product Test: (Not used anywhere anymore) The unlawful act was the product of mental
disease or defect (meaning, the disease made me do it).

Page 20 of 36

(d) Diminished Capacity: Relates to the defendants mental condition at the time of the offense, and its impact on the
defendants capacity to entertain the required mental state. (Negates mens rea element of a crimemitigating the
charge to a lesser crime.)
a. Test Asks: Whether the mental illness prevents the D from having the required mens rea for the crime.
(Minorityvery few jurisdictions use it---Florida doesnt)
(e) Abandonment:
a. General Rule: There must be a complete and voluntary renunciation of criminal purpose.
i. Completenot postponed.
ii. Voluntarytrue change of heart and not a change of plans caused by external forces.
b. Inchoate Crimes:
i. Florida: **Followed by MPC**
1. Attempt: D must abandon the attempt under circumstances manifesting a complete
renunciation of criminal purpose or otherwise prevent commission of the target crime.
2. Solicitation and Conspiracy: D must persuade the other person not to commit the crime,
or otherwise prevent the crime under circumstances manifesting a complete renunciation
of criminal purpose.
3.
ii. Non-MPC states:
1. Solicitation: Abandonment is not a defense except in jurisdictions that recognize
attempted solicitation (Not a universal rule but some states will hold that it cant be
abandoned because it is complete when made.)
iii. Common law: Abandonment was not a defense because the inchoate crime was complete when
done or attempted.
o Most states however, apply the complete and voluntary renunciation approach and just
customize it.
c. Complicity (Accomplice Liability):
i. MPC: Allows an accomplice, if D terminates his complicity before the target crime is committed
and D either:
1. entirely deprives his complicity of its effectiveness; or
2. warns law enforcement or otherwise makes a proper effort to prevent the crime.
ii. Florida: Same as MPC but adds that in homicide cases, D must also show that he clearly
communicated his renunciation to his accomplices or co-felons with sufficient time for them
to consider abandoning the crime.
(f) Impossibility: D admits to committing all elements of a crime but argues that he should be excused because the
crime was impossible to achieve. **MPC and Florida have abolished impossibility along with other states**
**Not a defense to conspiracyi.e. when a law enforcement officer makes the target crime impossible**
a. Factual: Occurs when circumstances unknown to the actor or beyond his control, prevent the
consummation of the crime, and is never a defense to attempt in common law, and not now.
i. Ex: the woman who tried to steal the organizer from his friends pocket, when pocket had a hole
and the organizer had slipped through it.
b. Legal: means that D made a mistake about the legal aspect of the offense that made it impossible to
complete the crime. Depending on the situation it could provide a defense to attempt.
i. Pure legal impossibility: D is guilty if he purposefully engages in conduct that would be a crime
if circumstances were as D believed them to be. (Ds conduct isnt illegal but he thinks he is
committing a crime.) **VALID DEFENSEFollowed by Florida and MPC because it is a Q
of Due Process** If what D is attempting to do isnt a crime you cannot charge him with a crime.
Ex: D thinks aspirin is illegal drug so he goes to score some from a drug dealer.
ii. Hybrid legal impossibility: When the actors goal is illegal but commission of the offense is
impossible due to a factual mistake (not simply a misunderstanding of the law) about the legal
status of some circumstances that constitute an element of the offense. **NOT ON EXAM**
It is not a defense to attempt in any jurisdiction, because legal impossibility is not a
defense if the crime could have been committed had the attendant circumstances been as
the actor believes them to be.
Ex: Case of the buyer of stolen goods, in which the employer discovered that the
employee was stealing from him to sell the goods to the buyer, and ordered another
employee to deliver a pending good to the buyer, without the buyer having knowledge of
it. The buyer had an illegal goal (buy stolen goods), but commission of the crime was
Page 21 of 36

impossible because when the employer consented to deliver the goods, they lost their
stolen legal status (the buyer was mistaken that the property was stolen).
Ex: People v. Dlugash, D shot dead body, he intended to kill someone but didnt know
guy was already dead, The court held him liable for attempt because if the circumstances
had been as he thought to be, he would have killed the victim.
Ex: Howard plays a joke to Dennis, and asks him to steal his new car from Carol, who
he lend his car previously. In this case, Howard would not be liable for solicitation because
it was his car. If Dennis would have stolen the car he would not be liable for theft because
he had the owners consent. However, he could be charged of attempted theft, because if
circumstances would be as he though them to be, he would have committed a theft. But If
Dennis would be in Florida, he would be charged of theft, because thats the charge for
attempt to commit theft.
c. Inherent: D uses a de minimus conduct to accomplish the target crime. (the means chosen by D are
manifestly unlikely to achieve the ends sought.) **Courts may use today**
i. Ex: Using a slingshot to bring down an airplane. D completed all the elements of the target crime
but it is impossible to bring down an airplane with that type of instrument.

Inchoate CrimesCrimes that are Anticipatory, incomplete, or failed crimes; allowing the police to intervene
and incapacitate criminals prior to the commission of a target offense.
(a) Solicitation:
o General Rule: A person is guilty of solicitation if he intentionally invites, requests, commands, or
encourages another person to engage in conduct constituting a felony, or a misdemeanor. (Solicitee must
be asked to be the primary actor and not a helperhelpers are generally accomplices.)
o Actus Reus:
o Verbs such as invitees, requests, induces, entices, persuades, etc.
o Involves speech or some communicative act.
o Does not include a general solicitation to a large, indefinable group.
MPC: Uses intentionally, encourages, and facilitates. Usually words considered as
complicity words. **BE CAREFUL, in some cases complicity words may constitutive
solicitation words as well**
o Effective once it is made. **Followed by MPC, Florida, and Majority Rule**
o Solicitation must be received by solicitee. Also recognizes attempted solicitation. (Minority view)
o Innocent Agent: If D deceives an innocent agent into participating in a crime, it still may be
solicitation. **Recognized by most states as an attempted solicitation**
MPC, recognizes this as a substantial step for attempted solicitation.
EXAM TIP: If you see a solicitation of an innocent agent in an MPC state,
consider that the crime could actually be an attempt for the solicitor.
Ex: State v. Bush, where D tricked innocent agent into thinking she was hired as a
photographer, and instead smuggled cocaine inside her camera bag. Ct held that the
essential part of the offense of solicitation is the intent of the solicitor and not the
knowledge of the victim.
o Mens Rea: Specific intentD must:
o Intend to commit the actus reus of solicitation (i.e. request, encourage, invite, another to commit a
crime); and
o Intend for the solicitee to commit the target offense. (even if target crime if a general intent crime)
Mens rea of solicitee is irrelevant; all that matters is the mens rea of the solicitor. (i.e.
State v. Bush)
If Solicitor is making a joke or being sarcastic then D doesnt have the requisite
mens rea for solicitation.
o Merger Doctrine: Solicitation merges with conspiracy, attempt and the completed target crime.
o Defenses: Abandonment is no defense. (once the solicitation is made, there is retracting it.)
o Exceptions: **Followed by MPC and Florida**
Renunciation: D must
Completely and voluntarily renounce his criminal intent; and
Page 22 of 36

Persuade the solicited party not to commit the offense or otherwise prevent him
from committing the crime.
(b) Attempt: Occurs when a person, with the intent to commit a criminal offense, engages in conduct that constitutes
the beginning of the perpetration of, rather than mere preparation for, the target offense.
o Applies to 3 factual situations:
- D tries to commit a crime but fails
D either misses entirely or hits but V survives.
- D is in the process of committing a crime but is intercepted
i.e. by police.
- D makes a mistake about something and that mistake makes it impossible to commit the target
crime.
Ex: D wants to have sex with a 14 yr old. Goes out and looks for one, finds someone who
looks 14 but is actually 21. Has sex and later realizes she was 21. D is still punished for
attempted statutory rape.
o Actus Reus:
o An overt act beyond mere preparation (can be minimal conduct, need not be illegalFL requires
ANY act.)
o Tests applied to determine if conduct satisfies the actus reus requirement: (tests that ask)
How close to completion was D?
1. Last Act: Criminal attempt occurs when D takes the last step needed for the
commission of the target offense but missed or failed. (Used by Majority)
o i.e. D shoots but misses V or gun jams.
2. Physical Proximity: Ds conduct must be "proximate" in time and space to the
completed crime. D must have the apparent power to complete the crime
immediately.
o i.e. Rizzo, where Ds drove around looking for their intended victim, who
didnt show up. Ct held that D didnt have the power to commit the crime
immediately because he had yet to find his victim.
o EXAM TIP: If fact pattern indicates someone driving around then apply
physical proximity test.
3. Dangerous Proximity: Attempt occurs when Ds conduct is so near the result that
the danger of success is very great. The more serious the offense, the less close the
actor must come to completing the offense to be convicted of attempt.
o Ex: Rizzo, because they lacked the ability to complete the crimethe
victim was missingthe court concluded that they were not yet
dangerously close to success.
o Hypo: Having a loaded gun and waiting in the bushes to shoot the V when
he got home. (but going to the gun store to buy a gun is not)
4. Indispensable Element: Attempt occurs when all elements necessary for
completion of the crime are under Ds control.
o i.e. there is no attempted assault w/ a deadly weapon if D still doesnt
have the weapon. Or if the indispensable element is the help of an
accomplice that had not yet arrived.
How close to beginning was D?
Probable Desistance: Attempt occurs when D reaches a point where a lawabiding citizen would probably desist. Focuses upon whether D, without
intervention by others, would have been likely to continue with the actions
necessary to complete the crime.
o Ex: D was found guilty of attempted lewd of lascivious acts when he
enticed a minor to his home for sexual acts and then met his intended
victim outside as the minor stepped out of the taxi. Ct held that D passed
the point where the law-abiding citizen would have turned back.
Abnormal Step: Attempt occurs when D takes an abnormal step at the point
where a law-abiding person would probably turn back. (no reported cases)
o Ex: Staples, D drilled holes in the floor above a bank.
Page 23 of 36

o
o

Res Ipsa Loquitor (unequivocality): Attempt occurs when Ds actus reus is so


unambiguous that an observer would guess what he intended to do. Liability
attaches at the point where an observer, knowing only of D's actions without
knowing Ds intent, would believe that D intended to commit the offense.
o No reported cases in USNew Zeeland case of people trying to steal a
car battery from another car after theirs died.
Substantial Step (MPC): D must take substantial step that is strongly
corroborative of Ds criminal purpose to commit the target crime.
o 5.01 (2) Lays out requirements for substantial step.
Mens Rea: D must have
The intent to commit the actus reus; and
the specific intent to commit the target crime (even if target crime is general intent crime).
Florida, rejects this approach and hold that mens rea for attempt follows the target
crime. (If crime is general intent, then mens rea or attempt is the same as general
intent crime.)
MPC, requires that D have
o The mens rea regarding the target crime
Requires the same as the target crime.
o The mens rea regarding the actus reus used to commit the target crime.
Must be purposeful. Occurs in 3 situations:
Ds conduct is purposeful but completed crime is
impossible.
o i.e. D shoots a tree stump thinking its a person.
**MPC abolished defense of impossibility**
Ds conduct is purposeful and thinks he has done all
necessary but something is missing.
o i.e. shoots but gun wasnt loaded
D has purposefully engaged in a substantial step towards
the target.
o Ex: Hypo where D is an environmentalist and
plans to kill the CEO of company planning to
drill holes in wilderness area. She buys the gun,
sits in wait, loads the gun, etc.
Merger Doctrine: Attempt merges with the completed target crime and with conspiracy. But D
cant be convicted of both Attempt and Target/Conspiracy.
Special Attempt Rules:
Attempted Felony-Murder: Most states consider this to be impossible. However, Florida
passed a special statute. (782.051) Elements are:
D committed or attempted to commit a predicate felony.
While committing, attempting or escaping from it, D either:
o (a) Committed or aided and abetted a separate intentional act that was not
an essential element of the predicate and could have caused death.
i.e. D sets a fire while escaping from a robbery so that the smoke
could distract and slow down his pursuers. Fire is not an essential
element of the predicate of robbery but it could cause death.
o (b) Victim was actually injured by a 3rd party
o (c) Predicate need not be enumerated felony, but if it is then D will be
liable for a higher degree of attempted felony murder.
Attempted Reckless Murder: (General intent crime)
Most states will not recognize the attempt (i.e. Leiola).
Florida does recognize this crime because the crime is Recklessness PLUS and
indifference to life.
Attempted Reckless and Negligence Crimes:
No state recognizes the attempt to commit a crime of negligence.
Page 24 of 36

Most states and the MPC dont recognize attempted recklessness for result crimes.
o Dont recognize Attempted Manslaughter
o Florida, doesnt recognize Attempted Manslaughter by Negligence
Those are crimes that require recklessness as to the result.
Attempt to Aid (MPC Provision): A defendant who acts as an accomplice may be
convicted of attempt, even if the principal party never commits or attempts to commit the
target crime.
Attempted Strict Liability: Occurs when D has specific intent to commit the actus reus
for the target crime. (i.e. attempted statutory rape. D only needs the specific intent to
commit the actus reus of sexual intercoursethe age element is a strict liability element so
what D knows or thinks about the age doesnt matter.)
Endeavor Statutes: (Florida Theft Statute) If D endeavors to steal; the crime is a
completed theft not an attempted theft. (D cant be charged with attempted theft in
Florida.)
(c) Conspiracy (based on the idea that group criminality is more dangerous than an individual criminality)
o General Rule: An agreement between 2 or more persons to commit an unlawful act or series of unlawful
acts.
o Elements:
- D must agree to commit a target crime; and
- Do so with the specific intent to have target crime succeed.
o Actus Reus:
- An expressed or implied agreement between 2 or more persons.
Some jurisdictions add an overt act with very minimal requirement.
Florida doesnt require an overt actjust minimal act.
MPC requires it unless it is for a 1st or 2nd degree crime (so more serious crimes
dont require it).
o Mens Rea: Specific intent for the target crime to succeed. (So, impossible to conspire to commit reckless
murder or involuntary manslaughter because they require recklessness as to result.)
- Some states recognize conspiracy to commit a crime that requires reckless as to conduct. Such as
reckless endangermentit is possible to conspire to commit this.
- Can also apply to crimes that have strict liability regarding circumstances.
o Ex: Feola, Victims age as a federal law enforcement officer or the age of a statutory rape
victim.
o Special Conspiracy Problems:
- Whether state requires bilateral or unilateral agreement.
o Common law and Florida, requires a bilateral agreement (at least 2 ppl for agreement)
o If 3rd party is a part of the conspiracy, then the presence of an undercover agent or
confidential informant will not create an invalid, unilateral conspiracy.
o MPC recognizes unilateral conspiracywhere it takes 2 people to agree but only one
person to be guilty of conspiracy.
- Lawful Suppliers: (Lauria case) If supplier knows that his goods are being used unlawfully,
knowledge is not enough to satisfy the mens rea for conspiracyhowever mens rea can be
inferred if the supplier charges an excessive price to illegal users, does an excessive volume of
services with illegitimate users, or there is no legitimate use for the goods or services.
o **Lauria rule, only applies to less serious crimes**. For more serious crimes knowledge
may satisfy conspiracy.
- Pinkerton Rule: Co-conspirators are liable for the crimes committed in furtherance of the
conspiracy, by the other co-conspirators, if they are reasonably foreseeable as a natural
consequence of the unlawful agreement. (A form of strict liability)
o MPC doesnt recognize it but Florida does (although they dont call it the same).
o Structure of Conspiracies: The Pinkerton rule liability is a huge advantage that results
from proving that multiple parties are engaged in one conspiracy (either through chain or
wheel), but parties must be aware of each other or of the overall scheme in order to be
linked to each other in the chain.
Page 25 of 36

Chain Conspiracy: Where each person or group in the conspiracy has specialized
responsibilities that link together the various aspects of the unlawful conductas
to create a community of interest among the parties.
Wheel Conspiracy: May look like a wheelin the center (the hub) is one person
or group, who/which conducts illegal dealings with various other persons or
groups (the spokes). In order to connect the spokes (put the rim around the wheel)
there must exist some community of interest between the spokes. (If no rim, then
you would just have a lot of smaller conspiracies.)
- Whartons Rule: If a crime, by definition, requires 2 or more people as willing participants, there
can be no conspiracy to commit that offense if the only parties to the agreement are those who are
necessary to the commission of the underlying offense. (i.e. Bigamy, Buyer/seller of drugs
crimes cant be committed by one person)
o If a 3rd party is involved then the rule will not apply and prosecutor can charge all 3 for
conspiracy.
o MPC, doesnt recognize it.
o Florida, doesnt mention it by name and no case applying it but there are buyer/seller
cases where court says that conspiracy cant exist because both parties have different mens
reas and are not agreeing to the same target crime (i.e. one wants to buy and another sell).
- Length of Conspiracy: Conspiracy terminates when the target crime is complete or agreement is
abandoned (but never terminates because of impossibilityif D cant commit crime because police
defeat the purpose then D is still liable.
o Ex: US v. Jimenez Recio,
Merger Doctrine: Conspiracy does not Merge.
o Non-MPC States, D can be convicted of both conspiracy and target crime.
o MPC, conspiracy merges w/ target crime or its attempt, unless the conspiratorial agreement
contemplates further crimes that have not yet been committed or attempted. (if more work is left to
be done then no merger will occur)

Complicity/Accomplice Liability

This is NOT a crime. This doctrine holds criminally liable those people who do not personally engage in the
conduct proscribed by the criminal law, but assist or encourage those who do. It is also called derivative liability
because it is derived from the conduct of another, known as the principal or primary actor.
EXAM TIP: Start with the principals liability then go to the accomplice liability, because that liability is
derivative.
D is treated as if he or she committed the crime. So much so that Florida refers to both the accomplice and
primary party as principals.
The Primary party need not be convicted, but states must still prove the primary parties crime beyond a
reasonable doubt in the accomplices trial. If primary party is acquitted because of the excuse defense, accomplice
can still be liable.
Common Law:
i.
An individual is criminally liable as an accomplice if state proves the following 2 mental states:
i. D intentionally aids or encourages the other to commit the criminal act; and
ii. D had the mental state necessary for the crime actually committed by other
ii.
The common law classified accomplices, for purposes of punishment, in different categories:
- Principal (the actor): a person who aids the commission of the crime at the time it occurs. These
are referred today as accomplices, and are liable for the same offense as the principal.
- Accessory before the crime - who aids the commission of the crime. Also referred today as
accomplice, and liable for the same offense as the principal.
- Accessory after the fact: who aids the criminal after the crime has been committed. (No
derivative liability)
o Ex: hide a robber, to prevent him to be found by the police. Many jurisdictions treat it as a
separate crime with a lesser punishment.
(a) Elements of Derivative Accomplice Liability:
i. Actus Reus: An accomplice is a person, who, with the required mens rea, assists, aids and abets the
primary party in committing an offense. (Any degree of assistance is enough, even if it was not needed or
that the primary party didnt know someone aided him). There are 3 types of acts:
Page 26 of 36

Assistance by Physical Conduct:


o i.e. Providing a weapon to the primary party or driving the get away car
Assistance by Psychological Influence: encouragement
o Ex: Wilcox v. Jeffrey, where D attended a concert where one of the musicians that
performed was not allowed to do so by the immigration officer, circumstance known to D.
D was found to encourage the illegal performance by buying the ticket to the concert,
regardless that the musician would still have performed on Ds absence.
Assistance by Omission: when D has a legal duty to actmust be proven by the state. Although
presence alone does not make the witness an accomplice to the crime, an exception is made when
D fails to fulfill his duty to act. In that case, the omission would operate as consent and
contribution to the crime.
o Ex: State v. Walden, where the mother of a child witnessed her husband beating her child
and did not do anything. Her failure to act operated as a contribution to the crime.
Complicity does not require the contribution to be a cause in fact of the commission of the crime.
(Ex: Wilcox.)
Mere presence is not enough. We need presence plus some affirmative act or duty to act.
**Universal rule**
o Ex: V.T., where D was charged as accomplice for robbing his friends relatives camcorder
just because he was present while his friends robbed it and did not do anything. There was
no complicity because he did not encourage the commission of the offense.
o **However, presence alone can be enough in cases like the one on the subway, and the
gang.
Merger Doctrine: Complicity is the only inchoate crime that does not merge into the target crime.
There is no causation element. D is liable even if the primary party would have committed the
crime anyway. Because the inherent evil is the assistance itself and not the result.
Principals knowledge:
o Rule: The D need not know of the complicity, as long as it actually helps. (i.e. if D leaves
the front door open for a thief, then the are an accomplice if the thief enters from that door
although thief has no idea who left the door open. But if thief enters through the back
door, then no accomplice liability because the assistance has to actually have helped
further the crime.)
Conviction of the principal: is not necessary to prosecute the accomplice. The same applies if the
principal is not found, not prosecuted due to mental incapacity, acquitted, etc.
o EXAM TIP: If primary party does know, check our facts and see if there is an expressed
or implied complicity.
MPC: Actus reus is satisfied if D solicits, aids, agrees to aid, or attempts to aid in planning, or
committing a crime. (does not include the word encouragement)
o Solicitation also satisfied the actus reus of complicity.
**EXAM TIP: If you a problem in an MPC for solicitation, consider that solicitor
could possibly satisfy complicity as well**
o Attempts to aidmeans that D is liable even if primary party does use the assistance and
even if the primary party never commits or attempts to commit the crime. (i.e. undercover
officer, but accomplice provides some sort of assistance to that officer.
ii. Mens Rea: D must have both the specific intent to assist AND the mens rea required for the target crime.
Specific Intentrequires more than mere knowledge of the crime. (A purpose to assist is
required)
o Ex: Beaman case
Target Crime mens reaif crime is of recklessness or negligence, its possible for D to be
convicted for helping in that crime. **Majority and MPC view**
o Ex: Hopkins, Case of the woman that gave her car keys to a friend who was intoxicated,
and was charged for manslaughter for aiding and abetting the commission of a reckless
crime. The court said that her intent toward the act was to give the car keys to her friend
while being drunk, and her intent to the principal crime was to let him drive being
intoxicated. Charge was involuntary manslaughter, which requires a mens rea of
recklessness.

Page 27 of 36

Ex: Flayheart, 2 Ds were convicted as accomplices to negligent homicide, where


deceased was the victim of their neglect. This case taught us that where the state has
problems proving which party was the primary party, both could be charged and convicted
as accomplices for each other.
(b) Liability for Other Crimes: D is liable for any other crime that was a natural and probable consequence of the
primary partys target crime. When:
The additional offenses are the natural & probable consequences of the conduct that D did intend to
assist (even though D did not intent these additional offenses) AND
The principal committed the additional crimes in furtherance of the original criminal objective that D was
trying to assist.
**Followed by Florida, Majority of State. NOT MPCno vicarious liability as general principal**
(c) Who is not an accomplice?
A party who abandons the crime. Provided he or she follows the requirements of abandonment.
Someone who is the victim of the primary partys crime but provides assistance (i.e. for instance by paying
ransom for a kidnapping.)
Someone whos conduct is incidental to the target crime (i.e. Person who marries a bigamistnot guilty of
bigamy)
An escaped prisoner is not guilty of the primary partys crime of harboring an escaped prisoner,
obstruction of justice or accessory after the fact.
(d) EXAM TIPS FOR COMPLICITY:
If you think someone is an accomplice on an Essay, start with the primary partys liability and then the
accomplice liability will derive from that. If primary party didnt commit the target crime, then neither did
the D.
Under MPC primary party need not attempt or commit the crime in order for D to be an accomplice, but
under traditional jurisdictions, the primary party must attempt or commit the crime.
D is not liable as an accomplice if primary party has a justification defense, but is liable if primary party
had an excuse defense (personal just to that D).
Accessory after the fact: (Not a derivative crime because it occurs after the crime) Helps after the crime
and not involved in the actual commission of the crime.
o Florida 777.03recognizes family exceptions. Any assistance provided by a blood or marriage
relative, will not make them liable as accessories after the fact. (i.e. husband/wife, parent/child,
brother/sister, grandparent/grandchild.) Applicable only to 3rd degree felonies. Doesnt apply to
anything greater.
o MPC doesnt recognize this actual crime but recognizes it in a different waythrough obstruction
of justice.
Innocent Instrumentality: an innocent person used to carry out the crime is not considered an
accomplice, because he lacks the mental state to commit the crime, and the accessory can be treated as
principal, becoming liable for both, the crime, and the use of an innocent person. Ex: Political extremist
sends a letter containing anthrax to a government office and the mail carrier delivers the letter.
o

Theft (Specific Intent Crimes)


(a) Larceny (Common law): Trespassory taking and carrying away of tangible chattel possessed by a 3rd party with
intent to deprive person permanently.
a. Common Law Elements:
i. Trespassory (without consent or lawful justification)
If taking was consensual then it wasnt larceny at common law.
Ex: the woman who asks his friend to lend his phone to see it, and as soon as it is given to
her, she runs away with, there is not larceny because the taking was not trespassory but
consented.
Legal Fictions created when taking occurs after the owners consent:
o Larceny by Trick: When D obtains possession consensually but through trick or
deception. Deception vitiated the consent.
Elements:
Obtain possession of personal property of some value
Belonging to a 3rd party
Page 28 of 36

ii.

iii.
iv.

v.

By making a false promise which he has no intention to


perform, or by means of fraud, artifice, trick, or device
(vitiates consent).
In surrendering possession, the alleged victim did not intend
to transfer ownership; and
The actor obtained possession with the specific intent
deprive permanently.
Identical crime, but applies where taking appears to be lawful, but in
fact consent to taking was by trickery or deceit. Still trespassory
taking. (Ex: Case of the girl that borrows the phone and runs away
with it.)
o Breaking Bulk: When property is turned over to the bailee, bailee has lawful
possession of the package, but only custody of its content. So taking the package
is not larceny, but opening it would be considered breaking bulk and is trespass on
the contents.
Ex, US v. Mafnas, case where D was the bailee of moneybags for a
bank,and on a few occasions opened the bags and stole money. Ct held
that Ds breaking bulk constituted trespass and larceny because it was w/o
consent. Had there been consent to the content then it would have been
embezzlement.
o Custody is not possession: temporary custody of property for a limited purpose is
not possession, because the owner retains constructive possession of the item,
therefore if D keeps the property it would be trespass.
Taking (caption) and Carrying away (asportation)
Involved exercising dominion and control over the item. (Ex: Carswell, Ct defined it as
severing the item for another persons possession.) If item could not be taken, then carrying
away would not be enough.
Carrying away could be satisfied by the slightest, smallest movement.
o Ex: Carswell, case of the air conditioner in the hotel, which was detached and
moved a few inches from its location. Thieves were in control for an instant (took
it), and carried it away (moved it). But shifting the position of an object without
moving it from where it was found is insufficient asportation.
Both elements are required.
o Ex: case of the man that took the coat chained to a dummy. There was asportation
but not possession (control), so there was no larceny (only attempt).
Personal Property
Tangible Personal Propertynot real property, services, intangible (stocks, bonds, etc.).
Possessed by a 3rd party
Property had to be taken from someone with superior possessory interest. This is not a
crime of ownership, but possession.
An owner can steal property that he own. (i.e. unlawfully retrieve your item from a repair
shop.)
Right to Recapture from a Thief:
o Defense of recapture: The right to reclaim property from a thiefapplies only
when in hot pursuit to recover chattel.
o Claim of Right Defense: when there is a claim of right there is no larceny (good
faith belief that the property belongs to the actor, or he had permission to use it, or
that the property does not have an owner). This is an affirmative defense.
Ex: laptop sold, which was paid with a bounced check
With Intent to Permanently Deprive
Intent: Mens rea and actus reus had to occur at the same time. (i.e. D took property
unlawfully by borrowing w/o permission but intended to return it later. He later realizes
that he wasnt going to return it. D would not be committed of common law larceny
because actus and mens rea didnt occur at the same time.)
Page 29 of 36

Continuing Trespass: Legal fiction developed to remedy the adverse affects of


the common law larceny rule. Every minute that the thief has the property
wrongfully will constitute a brand new trespassory taking. Each continues on the
time line until it finally collides with the thiefs later decision to keep the property.
Permanent Deprivation: Focuses on loss to the possessorthus deprived of the
properties primary economic values. This includes:
o Taking a seasonal item temporarily, if the lawful possessor is deprived of the
primary economic benefit of the seasonal item when he needs it. (i.e. Pool filter)
o Taking of property that is fungible. If original can be replaced with a similar item.
(i.e. gasoline) There is permanent deprivation because the gas replaced would be
different gasoline containing distinct molecules.
o Taking of property that is good only for seasonal use (i.e. pool filter); or
o Taking of perishable property. (i.e. Flowers cut)
Conditional Intent can also satisfy permanent deprivation element. This is because an
intent to steal is established where the takers conduct creates a substantial risk of
permanent loss. (Ex: Davis, thief steals clothes with the intent to ask for a refund later. If
refund is not obtained he would keep the merchandise.)
o Resale to the lawful possessor
o Reward for finding the property
o Return of the property for a refund
(b) Larceny by False Pretenses: D obtains title by fraud or deception (versus larceny by trick which only covers
possession).
a. Elements of theft by false pretenses:
i. Defendant takes title to the property
ii. By making a false statement
iii. About a present or past fact
1. Also an omission when there is a duty to disclose
2. It cannot be based on false statements as to the future
3. Ex: case of the man who told a car seller he will drive a car to see if he liked it and then
come back to pay for it, but never intended to do that. He did that to get possession. The
court dismissed the case because a false promise will not suffice for false pretenses,
however fraudulent they may be.
iv. Defendants statement must actually be false or Defendant must belief the statement is false
1. Ex: the man who sold cubic zirconium thinking is was a carat ring that ended up being a
carat ring was liable for theft by false pretenses.
2. If defendant thinks is true, no matter how naive the pretense is, there is no larceny.
a. Ex: Case of the people selling the magic medical device, where the court
dismissed the charges because D tried to prove that he believed in the
qualifications of the device by showing that he spoke to scientists, and doctors.
3. Statements of opinion or puffery, which a reasonable person would know is not true does
not satisfy this element. Ex: dealership seller saying the car is the best in the world.
v. Defendant must intend to defraud the victim
vi. Victim must rely in the false statement.
1. Ex: Lorenzo case, where D switched price tags. Manager knew, so he could not have
relied, so there was no theft by false pretenses.
(c) Embezzlement: Fraudulent conversion of personal property, subsequent to the lawful entrustment. By someone
who acquired it lawful consensual possession by means of entrustment by a third person (lawful possession).
a. This was not a crime at common law, it was created to fill in loopholes in Larceny when someone took
property with consent and later kept the property (stole), which would not satisfy the common elements of
larceny, (no trespass), and there was no trick either.
b. Elements of Embezzlement: D has lawful title or possession, through entrustment, D converts the
property to its own use by interfering with property rights, and D intended to fraudulently deprive the 3rd
party of his or her rights to the property.
i. Entrustment
1. D acquires title or possession of property through entrustment (i.e. watch repair service,
banker, investment manager, probationer)
o

Page 30 of 36

2. There doesnt need to be a fiduciary relationship. All required was initial trust by one party
in another.
ii. Conversion
1. D converts a property to its own use by interfering with property rights.
a. D need not gain a financial benefit or any benefit at all. All that is necessary is that
D treat the property as his own, regardless if the actor uses it, destroys it, sells it,
or discards it.
i. Ex: State v. Archie, where D was put a device on his probation, he
removed it, damaged it and threw it out. The court found embezzlement
because D was entrusted with the device, which was even in writing, and
he destroyed it and discarded, which constitute appropriation because
when he did that he acted as the propertys owner, and he intended to
deprive permanently, regardless that he did not intend to keep it.
iii. Fraud
1. D intends to fraudulently deprive a 3rd party of his or her rights to the property.
a. Can be temporary or permanent deprivation.
(d) EXAM TIP:
a. Ask whether the D intended to pass title or possession.
i. If title is being passed either embezzlement or false pretenses analysis.
1. If lawfully embezzlement
2. If unlawfully False pretenses analysis
ii. If possession and not title Common law larceny or embezzlement
1. If lawfully embezzlement
2. If unlawfully Common law larceny
3. If consent obtained by deceit Larceny by Trick
b. Modern Law (discarded all larceny statutes and created theft)
i. If statute doesnt define the word property or doesnt provide rules for measuring the value of
property. Courts will reach back into the common law of larceny.
ii. Reaching back to the common law could mean that labor, services, and even electronic documents
are not considered to be property. Valuable papers, such as the computer print out of work product,
would be valued at market value, which is scrap paper.
1. Ex: Lunt case, there was a statutory theft crime but the word property wasnt defined or
way of measuring value. Court reached into the common law and found that nothing that
he had done amounted to larceny.
2. Ex: Farraj case, Ct held that an electronic document constituted goods rather than
information so it was covered by the wording of the statute. (This will always depend on
the wording of the statute so be careful)
iii. Floridas theft statute pretty much has it all covered. Even possession of stolen property is treated
as theft.
iv. MPC is the same as Florida but doesnt recognize attempted theft as a completed crime.

Arson
a) Common law Arson: Malicious and willful burning of the dwelling house of another. (Crime against habitation
rather than property.) Result crime at common law requiring causation.
a. Actus Reus:
i. Burning: The consuming of the material of the house, or burning of any part of the house.
1. Damage didnt have to be serious or extensive but the fire must have charred some part of
the dwelling. (Slight charring was sufficient, smoke was not enough)
2. Damage had to come from the burning and not damage from water pipe or anything
else. Explosion isnt enough either unless explosion causes fire and charring.
3. An explosion without causing fire was not arson, fire causing water damage, was not
arson, setting personal property in fire is not arson either.
ii. Dwelling House:
1. Included the house where occupant actually lived and any buildings in the curtilage (i.e.
barn, shed, etc.) Setting fire to those would be considered part of the occupied dwelling.
The concern was with life and not property.
Page 31 of 36

2. If house is rented, the tenant has the right to burn it but the owner does not. That is
because the right protected is the habitation right.
3. Building had to be actually occupied as a dwelling. If they moved out or abandoned the
property, there would be no arson. Temporary absence would not make a dwelling be
considered unoccupied. The concern is that it be occupied at the time.
4. Hotels, jails, hospitals, and other structures where there would be multiple dwellings were
not considered occupied unless someone lived there as a permanent resident. Jail could be
considered an occupied dwelling if warden lived there.
iii. Of Another person
1. Applies not only to owner of property but also to renters.
2. Owners had the right to burn their own property (not considered arson). If owner burns the
house for insurance money is not arson, although insurance fraud.
b. Mens Rea:
i. Willfulness and malicious intent to burn (both are neededproven separately)
1. Willfulness meant that arsonist must have started the fire intentionally.
2. Malice meant D either (a) intended to burn the building (b) knew that his act would burn
the dwelling, or (c) intentionally created a fire hazard that threatened to burn the building.
3. This is a general intent crime; even though it is a result crime, the D does not need to
specifically intend the damage, just the intent to start the fire.
c. Resulting Damage; and
d. Causation
e. Defenses:
i. Intoxication is not a defense.
ii. Mistake of fact had a more heightened requirementHad to be an honest AND reasonable
mistake of fact.
b) Modern Statutes: No longer limited to charring. As long as D starts a fire with intent to damage the structure he
can be convicted even if no damage or charring. Have also expanded the statute to include explosion damages even
if it doesnt involve fire.
c) Florida Arson: D commits arson in Florida when he or she damages or causes to be damaged a structure or its
contents, by fire or explosion, and does so (a) willfully and unlawfully, or (b) while engaged in the commission of
a felony.
a. Elements:
i. Defendant damages or causes to be damaged;
1. There is no difference of discoloration, water damage, etc.
ii. A structure or its contents
1. Not limited to dwelling.
2. Structuremeans a building with any enclosed area and a roof, any real property and
attachments, tent portable building, water vessel, vehicle, aircraft.
3. Dwellingalso includes curtilage, dwelling, and attached dwelling.
4. 1st degree Arson: If dwelling, or structure with high probability of occupancy (hotel or
office buildingmore serious when life involved)
5. 2nd degree Arson: If its not a dwelling or a structure with high probability of occupancy.
a. If the home is severely damaged by hurricane that took the roof off, or somehow
legally affected its being categorized as a structure, it would still be recognized as
a structure for purposes of arson (second degree arson)
6. There is no agreement in Florida as to whether the property is considered a structure for
arson purposes when the property is inhabitable because of reconstruction, etc.
7. There is a difference between inhabiting and occupying (homeless).
iii. By fire or explosion;
iv. Either willfully or unlawfully or while committing another felony, (felony murder, where mens
rea is borrowed from other crime).

Rape
a) Common Law Rape: Carnal knowledge of a woman forcibly and against her will.
a. Elements of common law rape:
i. Carnal knowledge of a woman,
1. Sexual intercourse only
Page 32 of 36

b)

2. Slight penile penetration was enough


3. Rape with object or anal rape was not included, and could only be performed against a
woman, not his wife.
4. Gender specific definition requiring a male D and female Vbut female could be
convicted as accomplice for assisting the man with the commission of the rape.
ii. Forcibly, and
1. Physical force or threat of force likely to cause serious bodily injury to woman or her
children.
2. Non-physical force, such as economic threat, would not be enough.
3. Age of consent was 10. Over the age of 10, consent was presumed.
a. Prosecution had to prove lack of consent as an element at common law by
showing that the woman resisted.
iii. Resistance: Woman had to resist as long as she could and to the point of exhaustion.
1. Ex. People v. Warren, where D lifted V and took her someone where secluded and raped
her without force or threat. V claimed that she submitted cause she was afraid he would
kill her. Ct held that Vs actions showed no resistance. Gave 3 circumstances where V
doesnt have to resist: (1) when it is pointless to do so, (2) life threatening, or (3) V was
overcome by superior strength or power. Non-existed here cause D never used his size or
strength or physical force.
2. Marital exemption: Man could not be convicted of raping his wife. But can be guilty of
accomplice if he allowed others to rape his wife.
Modern Rape Statutes: Have been renamed sexual assault and sexual battery.
a. Generally divided into degrees in terms of the seriousness of the assault, extend of assault, age, mental
disability, consciousness, etc.
b. Are now gender neutral.
c. In modern times, any physical force used against the victim qualifies, even though it entails no injury and
leaves no mark.
d. Some states also consider coercion by other threats, like threat to retaliate by public humiliation, duress,
etc.
e. MPC
i. Does not recognize same sex rape.
ii. Has retained the marital exemption for spouses and couples living together as husband and wife
and uses a lesser felony for date and acquaintance rape.
iii. Does not require resistance as an outer manifestation of consent. While the showing of resistance
may help to prove the issue of force or non-consent, its absence may not.
f. Florida
i. Marital exemption has been abolished. (majority of states too)
ii. Does not require resistance as an outer manifestation of consent. While the showing of resistance
may help to prove the issue of force or non-consent, its absence may not.
iii. Consent: (sect 794.011(a)
1. Intelligent, knowing, and voluntary consent (not including coerced)
2. Evidence of mental capacity can influence consent.
3. Crime is more serious if offender is in a position of trust (employer, or family guardian)
4. Has rule that a victims request that an attacker use a condom is not consent.
g. Elements:
i. Actus Reus:
1. Include all forms of sexual penetration.
a. MPC only genital, oral or anal penetration.
b. Florida does same as MPC and includes sexual union. (sect 794.011)
2. Trend has been to abolish resistance requirement. Even those states that retain the
requirement dont require more than reasonable under the circumstances.
3. Age of consent is higher than common law. MPC still has age 10 although no MPC state
has adopted that age.
4. Against her will
a. Lack of consent is enough
b. Consent was presumed absent physical manifestation of resistance
Page 33 of 36

i. Ex: Case of the girl compelled to perform oral sex on an unknown person,
where the court found consent for lack of physical force or threat of use of
a weapon, and lack of resistance, when girl didnt try to avoid the
encounter or said no to the stranger.
c. Lack of consent must be proven under an objective standard. The mental state of
the victim is irrelevant.
d. Modern approaches:
i. This element is established if prosecution must prove either that the
defendant knew that he did not have permission, or that his belief that
there had been consent was unreasonable under the circumstances.
ii. A minority of jurisdictions that retain the requirement of resistance,
require only as much resistance as is reasonably under the circumstances
or that is sufficient to indicate that the intercourse was not consented.
iii. A extreme minority of jurisdictions, non consent is presumed.
1. Ex: M.T.S. the court found she was not sleeping, but still found
that she did not consent. The only physical force required was that
necessary for an authorized penetration.
iv. Withdraw of Consent: If V changes her mind during sexual intercourse,
the act is not criminal (minority). Consent can be withdrawn at any time
(majority)
ii. Mens Rea (General Intent crime): only mens rea required is the intent to engage in the conduct
without consent, because by definition, a woman who did not consent must have been forced.
1. Defendant is not required to have any further intent such as inflicting an injury
2. Element of consent is a STRICT LIABILITY element (Majority)even an honest
mistake of fact as to consent is not a defense.
3. The issue for the courts is if the victim actually consented, and not if the defendant could
reasonably believe that victim consented.
a. This could cause problems in the case that the defendant is drunk, which will be
irrelevant to his intention to penetrate, but it could be an issue in the mens rea
regarding the victims consent, because due to his drunkenness, he may actually
appreciated that the victim consented.
b. However, in many states the mental state for the victims consent is strict liability,
so not even an honest and reasonable mistake of fact is a valid defense for lack of
consent.
c. The state only needs to prove:
i. That defendant intended to have sexual intercourse by force; and
ii. That the victim did not consent.
1. It does not matter what the defendant though about her consent.
d. It is immaterial at what point the victim withdraws her consent, so long as that
withdrawal is communicated to the male and he ignores it.
iii. Fraud as an alternative to force: the traditional law of rape allowed it in only two circumstances:
1. When penetration occurred during a medical examination, while a doctor made the
woman believe he was using a medical instrument, or while the patient was numb, or
2. When defendant deceived the woman into believing he is her husband.
iv. Fraud by deceit (to persuade) is not a substitute to force when the victim knew she was
consenting to sexual intercourse. Ex: woman that consents to have sex to get a job.
h. MPC: (very archaic) (woman that is not his wife)
i. Levels of felony rape:
1. Second degree: when a male have sexual intercourse with a female, not his wife, by
force or threat of imminent death, serious bodily injury, or extreme pain, or kidnapping to
be inflicted on anyone; or
a. He was drugged the victim without her knowledge to prevent resistance;
b. The female is unconscious; or
c. The female is under 10 years old.
2. First degree: Elevates Second-degree felony rape, where D:
a. inflicts basic act of rape and actually inflicts serious bodily injury upon anyone;
Page 34 of 36

b. used drugs or intoxicants on the female to prevent her from resisting;


c. V is unconscious; or
d. V is less than 10 years old. or anyone, or the victim was not his voluntary social
companion who has previously had sexual intercourse with him.
3. Third degree (gross sexual imposition): When a man compels a woman, not his wife, to
submit by any threat that would prevent resistance by a woman of ordinary resolutions;
or:
a. He knows she suffers a mental disease of defect that affects consent and renders
her incapable of appraising the nature of her conduct;
b. By fraud, D he knows she is unaware that sexual act is being committed upon her;
c. D knows that she submits because she mistakenly believes that he is her husband;
or
d. V was his voluntary social companion on the occasion of the crime and previously
permitted him sexual liberties. (Date or acquaintance rape)
ii. Mens Rea: D knowingly, purposefully, or recklessly compels the female to have sexual
intercourse. (negligence not included)
1. About the mens rea regarding the victims consent, (what D though about victims
consent), mistake of fact is allowed as a defense, unless the defendant is reckless.
2. Ex: if victim was intoxicated, then defendant was reckless in not appreciating her state of
intoxication, so no mistake of fact to consent can be argued.
3. Mistake of Fact Defense: Allows this defense unless D was reckless as to whether the V
was consenting.
iii. Highlights of MPC rape statute:
1. It excludes consent as an element of the crime
2. Is gender specific (only a man as the D and a woman as the victim
3. Retains spousal immunity (a man can rape his own wife)
4. It mitigates punishment when the victim is a voluntary social companion of defendant, and
had engaged in sexual intercourse with him before.
5. Threats allowed to vitiate consent are broader.
6. Mistake of fact is allowed as a defense, unless the defendant is reckless.
i. Florida: repealed rape statutes and substituted for sexual battery, which occurs in 3 situations:
i. Oral, anal, or vaginal penetration by sexual organ;
ii. Oral, anal, or vaginal union
1. Any touching or contact with the sexual organ);
iii. Anal, or vaginal penetration by object, which requires entry into the body, but it could be very
slight.
1. Penetration for bona fide medical purposes is exempt.
iv. As most modern sexual battery statutes Floridas is gender neutral both as to the perpetrator and
the victim. Therefore it includes homosexual rape.
v. Florida does not require resistance
vi. Only requires no more force than is required for penetration but express consent not required
(majority)
vii. Lack of consent must be proven as is an element of the crime, but if the victim is under 12, lack of
consent is an irrebuttable presumption (lack of capacity to consent).
1. Factors that can influence the consent determination are:
a. Mental incapacity or defect of the victim
b. Age of the victim, when under 18 and D is a relative or custodial authority,
acquiescence is not consent
c. When D is in a position of control or authority regarding the victim, acquiescence
is not consent. Ex: cops.
viii. Intoxication of defendant is not a defense, and a request for defendant to wear a condom does not
imply consent by the victim.
c) Statutory rape: creates strict liability rape when the victim is a minor.
a. Modern Statutory Approach:
Female victim under the age of 12 (punished as if it were forcible rape)
Female victim over the age of 12 (lesser felony depending on how much older)
Gender neutral
Page 35 of 36

Age is calculated by birth date. D or V becomes a certain age immediately after midnight on the
anniversary of birth. (i.e. born on 12/1/1997 at 3pm, turns 16 on 12/1/2016 at 12.01am.)
b. Florida: (Strict liability crime when sexual intercourse occurs with a minor)
Forcible Rape: V is Less than age 12irrebuttable presumption that he or she does not have the
legal capacity to consent.
Unlawful sexual activity with social minors: When D is 24 or older, and victim is 16-17. N/a to
emancipation.
Lewd or Lascivious battery: When victim is 12-16, Ds age is irrelevant.
Vaginal or oral penetration by, or union with, the sexual organ of another. Excludes an act
done for bona fide medical purposes.
Romeo and Juliet Laws: Registry doesnt distinguish between young sex offenders who have
consensual sex, and those who harm children and pose a real risk to society. They all end up on the
registry. Florida law allows a young offender, convicted of statutory rape, to file a petition to
remove the requirement to register as a sex offender under the following circumstances:
D must be at least 14 years old
Must be no more than 4 years older than V at time of crime
V must have consented in fact to sexual conduct. (not legal consent but voluntariness
consent)
c. Mens Rea Approach Among Jurisdictions:
MPC
Recognize mistake of fact as to age as a defense because it requires D to know the
victim is underage, and lack of knowledge negates that element. (Some other states too
developing trend) Must show that he did everything possible to avoid it being statutory
rape.
MPC requires D to be less than reckless about finding out that fact, and only applies when
the victim is of an age other than 10.
Mentally retarded person can reasonably believe the victim is older.
Florida
Do not recognize mistake of fact defense. Element is a strict liability.
o It is irrelevant that the victim misrepresented his/her age.
Silence of the statute about the mens rea would make courts to follow either common law or
majority approach. But strict liability needs to be expressed not implied.
Consent of the victim is not a defense because the actus reus is prohibited by law.

Page 36 of 36

Vous aimerez peut-être aussi